You are on page 1of 46

tomach and Esophagus MCQ

1. Which of the following statements about the anatomic course of the esophagus is correct?
A. The cervical esophagus passes behind and to the right of the trachea.
B. The thoracic esophagus enters the posterior mediastinum anterior to the aortic arch.
C. The thoracic esophagus passes behind the right mainstem bronchus and the pericardium.
D. The esophagus enters the diaphragmatic hiatus at the level of T8.
E. The esophagus deviates anteriorly and to the left as it enters the abdomen.
Answer: E

DISCUSSION: Knowledge of the normal course of the esophagus is important in operative and endoscopic
procedures. The cervical esophagus lies just anterior to the prevertebral fascia and courses through the
neck posterior to the trachea and to the left of the midline. The cervical esophagus is therefore more
readily approached surgically through a left neck incision. The thoracic esophagus enters the posterior
mediastinum posterior to the aortic arch. Therefore, when operating on the upper esophagus, a right
thoracotomy is preferred, since the aortic arch overlies the esophagus in the left chest. The esophagus
descends in the posterior mediastinum behind the left mainstem bronchus and pericardium. It is the
proximity of the esophagus and left mainstem bronchus that is responsible for the development of
malignant tracheoesophageal fistulas between these two structures. The esophagus enters the
diaphragmatic hiatus, which is located at the level of T11. As it enters the abdomen, the esophagus
deviates anteriorly and to the left. Therefore, when performing rigid esophagoscopy as the distal
esophagus is entered, the patient's head must be turned to the right and the esophagoscope elevated
anteriorly to avoid perforation.

2. Which of the following statements about esophageal anatomy is correct?


A. The esophagus has a poor blood supply, which is segmental in distribution and accounts for the high
incidence of anastomotic leakage.
B. The esophageal serosa consists of a thin layer of fibroareolar tissue.
C. The esophagus has two distinct muscle layers, an outer, longitudinal one and an inner, circular one,
which are striated in the upper third and smooth in the distal two thirds.
D. Injury to the recurrent laryngeal nerve results in vocal cord dysfunction but does not affect
swallowing.
E. The lymphatic drainage of the esophagus is relatively sparse, localized primarily to adjacent
paraesophageal lymph nodes.
Answer: C

DISCUSSION: Poor technique, not poor blood supply, explains most esophageal anastomotic leaks. While
the major blood supply of the esophagus is from four to six segmental aortic esophageal arteries, there
are extensive submucosal collaterals from the inferior thyroid, intercostal, bronchial, inferior phrenic,
and left gastric arteries. The esophagus lacks serosa and instead is surrounded by mediastinal connective
tissue (adventitia). There are two muscle layers in the esophagus, an outer longitudinal and an inner
circular one. Both layers of the upper third of the esophagus consist of striated muscle, while in the
lower two thirds they are (nonstriated) smooth muscle. The recurrent laryngeal branches of the vagus
nerves provide both parasympathetic innervation to the cervical esophagus and innervation to the upper
esophageal sphincter (UES). Injury to the recurrent laryngeal nerve therefore results in improved UES
function with secondary aspiration on swallowing as well as vocal cord dysfunction and hoarseness. The
esophagus has extensive lymphatic drainage, with lymphatic capillaries coursing longitudinally in the
esophageal wall and communicating with paraesophageal, paratracheal and subcarinal, other
mediastinal, perigastric, and internal jugular lymph nodes. This accounts for the biologically aggressive
nature of esophageal carcinoma, which tends to metastasize early in its course.

3. Which of the following statements about the lower esophageal sphincter (LES) mechanism, or high-
pressure zone (HPZ), is true?
A. The LES is a circular smooth muscle ring that is 3 to 5 cm. long.
B. In assessing esophageal manometric data, mean HPZ pressure less than 6 mm. Hg or overall length
less than 2 cm. is more likely to be associated with incompetence of the LES and gastroesophageal
reflux.
C. Esophageal manometry and the acid perfusion (Bernstein) test reliably identify the patient with an
incompetent LES mechanism.
D. Distal HPZ relaxation occurs within 5 to 8 seconds of initiating a swallow.
E. Twenty-four–hour distal esophageal pH monitoring is achieved with an intraesophageal pH electrode
positioned at the esophagogastric junction.
Answer: B

DISCUSSION: The LES is not an anatomic muscular sphincter like the anus or pylorus, but rather, is a
functional sphincter that serves as a barrier against abnormal regurgitation of gastric contents into the
esophagus and is more appropriately termed the distal esophageal HPZ. While, in general, no HPZ value
absolutely indicates either competence or incompetence of the LES mechanism, patients with a mean
HPZ pressure less than 6 mm. Hg or a sphincter length less than 2 cm. are likely to have an incompetent
LES and gastroesophageal reflux. Esophageal manometry defines the amplitude and length of the distal
HPZ and the character of esophageal peristalsis. It does not determine whether or not the LES is
competent. The Bernstein test identifies the patient with an acid-sensitive esophagus but does not
indicate whether or not the patient has gastroesophageal reflux or esophagitis. Reflex distal HPZ
relaxation occurs within 1.5 to 2.5 seconds after a swallow is initiated and lasts 4 to 6 seconds. Twenty-
four–hour distal esophageal pH monitoring is achieved by means of an intraesophageal pH electrode
positioned 5 cm. proximal to the HPZ, as determined by prior manometric evaluation.

4. Which of the following statements about esophageal motility is/are true?


A. The act of swallowing initiates UES relaxation, which persists until the bolus of food passes the LES.
B. The primary peristaltic wave normally propels the swallowed bolus through the esophagus in 4 to 8
seconds.
C. Normally, a progressive peristaltic contraction (primary wave) follows 50% of all swallows, the
remainder being secondary or tertiary contractions.
D. Secondary peristalsis is initiated when the entire swallowed bolus of food fails to empty from the
esophagus into the stomach.
E. Tertiary esophageal contractions are high-amplitude progressive peristaltic contractions that produce
the “corkscrew” appearance of esophageal spasm on barium esophagography.
Answer: BD

DISCUSSION: Swallowing initiates UES relaxation, which lasts only 0.5 to 1.0 second, considerably less
than the 4 to 8 seconds required for a swallowed bolus to pass through the esophagus and into the
stomach. A progressive peristaltic contraction normally follows 97% of all swallows. If the swallowed
bolus does not empty completely into the stomach, distention of the esophagus initiates secondary
peristalsis, which is progressive and sequential and persists until the retained intraesophageal contents
have passed into the stomach. Tertiary contractions are simultaneous, nonprogressive, incoordinated
contractions of the esophageal smooth muscle that produce the contraction rings responsible for the
corkscrew esophagus of diffuse esophageal spasm on barium esophagography.

5. Which of the following statements about UES dysfunction are correct?


A. This condition is diagnosed by the characteristic manometric findings of UES spasm.
B. Typical symptoms include cervical dysphagia, expectoration of saliva, and hoarseness.
C. The classic finding on barium esophagogram is a posterior cricopharyngeal bar.
D. Medical or surgical therapy of gastroesophageal reflux may be curative.
E. A cervical esophagomyotomy for UES dysfunction should be limited to 2 to 3 cm. in length so that
normal muscle is not damaged.
Answer: BCD

DISCUSSION: The unique anatomic characteristics of the UES and the limitations of existing equipment in
recording the rapid sequence of events associated with swallowing make standard manometric definition
of UES motor abnormalities extremely difficult. Characteristic consistent abnormalities of UES function
in patients with cricopharyngeal dysfunction have not been well documented. UES dysfunction results in
cervical dysphagia, expectoration of saliva that is no longer swallowed freely, and, often, intermittent
hoarseness due to alteration of the larynx and vocal cords by the pull of the abnormal cricopharyngeal
sphincter. Barium esophagography in the patient with UES dysfunction frequently shows a posterior
cricopharyngeal “bar,” representing the prominent impression of the sphincter on the esophageal lumen.
Patients with gastroesophageal reflux may present with cervical dysphagia due to secondary UES
dysfunction. Successful medical or surgical treatment of the reflux may eliminate the cervical
complaints. Since the UES is normally 3 to 5 cm. long, when a cervical esophagomyotomy is required for
treatment of cricopharyngeal dysfunction, a generous myotomy, 7 to 10 cm. long, is carried out to
ensure complete division of all incoordinated UES muscle fibers.
6. Which of the following statements about achalasia is/are correct?
A. In most cases in North America the cause is a parasitic infestation by Trypanosoma cruzi.
B. Chest pain and regurgitation are the usual symptoms.
C. Distal-third esophageal adenocarcinomas may occur in as many as 20% of patients within 10 years of
diagnosis.
D. Manometry demonstrates failure of LES relaxation on swallowing and absent or weak simultaneous
contractions in the esophageal body after swallowing.
E. Endoscopic botulinum toxin injection of the LES, pneumatic dilatation, and esophagomyotomy provide
highly effective curative therapy for achalasia.
Answer: D

DISCUSSION: While in South America achalasia is the result of Chagas' disease caused by parasitic
infestation by the leishmanial forms of T. cruzi, in Europe and North America the cause of achalasia is
unknown. The common presenting symptoms of achalasia are dysphagia, regurgitation, and weight loss.
Chest pain is an infrequent symptom in achalasia and is more characteristic of esophageal spasm.
Achalasia is a premalignant esophageal lesion: the retention esophagitis leads to metaplasia and
squamous cell carcinoma, which occurs after 15 to 25 years in the middle third of the thoracic esophagus
in 10% of patients. The classic manometric findings of achalasia are failure of relaxation of the LES on
swallowing and absent or weak simultaneous contractions in the esophageal body after swallowing.
Achalasia is currently incurable, and, though the recently described endoscopic botulinum toxin
injection of the lower esophageal sphincter, pneumatic dilatation, and esophagomyotomy effectively
relieve dysphagia in the majority of patients, all of these treatments are strictly palliative. The motility
disturbance persists throughout life.

7. Which of the following statements about diffuse esophageal spasm is/are true?
A. Chest pain due to esophageal spasm is readily differentiated from angina pectoris of cardiac origin.
B. Bouts of esophageal obstruction and regurgitation of food are characteristic.
C. Associated psychiatric disorders are common.
D. During manometric assessment, unless the patient is having pain there may be no detectable
multiphasic, high-amplitude, simultaneous esophageal contractions.
E. The treatment of choice is a long esophagomyotomy from the aortic arch to the esophagogastric
junction.
Answer: CD

DISCUSSION: The chest pain of diffuse esophageal spasm is often indistinguishable from that of angina
pectoris of cardiac origin. Many patients undergo cardiac catheterization to rule out coronary artery
disease. Patients may experience slow emptying of the esophagus, but obstructive symptoms and
regurgitation of food are unusual. Psychiatric disorders (depression, psychosomatic complaints, anxiety)
have been documented in more than 80% of patients with esophageal contraction abnormalities. The
classic manometric criteria of diffuse esophageal spasm are simultaneous, multiphasic, repetitive, high-
amplitude contractions occurring after a swallow and spontaneously. These changes may not be
detected if manometry is performed when the patient is having no pain. As the cause of esophageal
spasm is unknown, treatment is far from ideal. Conservative management—avoidance of “trigger” foods
and drinks, psychiatric counseling, treatment of reflux, esophageal dilatations, use of histamine H 2
blockers, anti-spasmodics, and smooth muscle relaxants—should always be attempted first.
Esophagomyotomy provides no lasting relief of esophageal spasm for as many as 50% of patients and
should be used only in a minority of these patients.

8. Which of the following statements about epiphrenic diverticula of the esophagus is/are correct?
A. They are traction diverticula that arise close to the tracheobronchial tree.
B. They characteristically arise proximal to an esophageal reflux stricture.
C. The degree of dysphagia correlates with the size of the pouch.
D. They are best approached surgically through a right thoracotomy.
E. The operation of choice is a stapled diverticulectomy, long esophagomyotomy, and partial
fundoplication.
Answer: E

DISCUSSION: Epiphrenic diverticula are pulsion diverticula that arise in the distal 10 cm. of the
esophagus. The cause is elevated intraesophageal pressure, which forces mucosa and submucosa to
herniate through the muscle layers. Though this may occur with an esophageal reflux stricture or tumor,
primary disordered esophageal motility is the most common cause. Many patients are asymptomatic
when their epiphrenic diverticula are diagnosed by barium esophagography, symptoms being related
more to the degree of disordered motility than to the size of the pouch. Distal esophageal diverticulum
is best approached through a left thoracotomy. When surgery is indicated, the preferred approach is
transthoracic diverticulectomy using a surgical stapler, a long esophagomyotomy to relieve the elevated
intraesophageal pressure, and a nonobstructing partial (e.g., Belsey's) fundoplication.

9. Which of the following statements about Schatzki's ring is correct?


A. The ring represents a panmural fibrotic stricture resulting from gastroesophageal reflux.
B. Dysphagia occurs when the ring diameter is 13 mm. or less.
C. The ring occurs within 1 to 2 cm. of the squamocolumnar epithelial junction.
D. Schatzki's ring indicates reflux esophagitis.
E. Schatzki's ring signifies the need for an antireflux operation.
Answer: B

DISCUSSION: Schatzki's ring is seen radiographically as an annular weblike constriction at the


esophagogastric junction in a patient with a sliding hiatalhernia. It represents prominence of the
esophagogastric junction with slight submucosal fibrosis but not true panmural fibrotic reflux stricture.
Intermittent dysphagia may occur when the ring size is 20 mm. or less, but dysphagia is almost invariable
when the ring measures 13 mm. or less. Schatzki's ring occurs precisely at the squamocolumnar epithelial
junction. It is indicative of the presence of a hiatal hernia but not of gastroesophageal reflux or
esophagitis. An asymptomatic Schatzki's ring requires no therapy per se. Patients with refractory severe
reflux symptoms after dilation therapy are candidates for antireflux surgery.

10. Choose the distance in centimeters from the upper incisor teeth at which the following
radiographically identified esophageal lesions would be encountered endoscopically:
A. 10 cm. 1. Zenker's diverticulum
B. 15 cm. 2. Traction diverticulum
C. 25 cm. 3. Tumor 10 cm. proximal to the esophagogastric junction
D. 30 cm.
E. 40 cm.
Answer: 1-B, 2-C, 3-D

DISCUSSION: A barium esophagogram should be obtained routinely before performing elective


esophagoscopy. The location of an esophageal lesion seen on the barium swallow study can be related to
adjacent anatomic landmarks. This allows the endoscopist to anticipate the level (as measured from the
upper incisor teeth) where he should expect to see the abnormality at esophagoscopy. For example, the
upper esophageal (cricopharyngeal) sphincter is generally seen at the level of the C7–T1 vertebrae
radiographically and at 15 cm. endoscopically. This is the level at which the mouth of a Zenker's
diverticulum is seen. The tracheal bifurcation occurs at the level of the T4 or 25 cm. from the upper
incisors endoscopically; a traction (parabronchial) diverticulum at the level of the carina on a barium
esophagogram will be seen at approximately 25 cm. The esophagogastric junction occurs at
approximately the level of T11, 40 cm. from the upper incisors; a tumor 10 cm. proximal to the
esophagogastric junction stricture is seen endoscopically at 30 cm.

11. Which of the following statements about pathology encountered at esophagoscopy is/are correct?
A. Reflux esophagitis should be graded as mild, moderate, or severe, to promote consistency among
different observers.
B. An esophageal reflux stricture with a 2-mm. lumen is not dilatable and is best treated with resection.
C. A newly diagnosed radiographic distal esophageal stricture warrants dilation and antireflux medical
therapy.
D. In patients with Barrett's mucosa, the squamocolumnar epithelial junction occurs 3 cm. or more
proximal to the anatomic esophagogastric junction.
E. After fasting at least 12 hours, a patient with megaesophagus of achalasia can safely undergo flexible
fiberoptic esophagoscopy.
Answer: D

DISCUSSION: The traditional subjective grading of reflux esophagitis as mild, moderate, or severe has
inherent wide variations in meaning among observers. Consistent use of standardized grading systems for
endoscopic reflux esophagitis (e.g., that of Belsey or Savary) provides a more objective description of
the changes seen and allows more meaningful evaluation of patients at different times and by different
observers. The size of the lumen does not predict whether or not a reflux stricture is dilatable. Even a
tight 2-mm. lumen can be traversed with a guidewire over which Savary dilators can be used to achieve
an acceptable lumen size. Every newly diagnosed esophageal stricture warrants esophagoscopy with
brushings and biopsies of the stricture (to exclude carcinoma) and an assessment of its “dilatability.”
Antireflux medical therapy is not justified until carcinoma has been ruled out. Because the
squamocolumnar epithelial junction may normally be found within 2 to 3 cm. of the anatomic
esophagogastric junction, the diagnosis of Barrett's mucosa requires identification of the columnar
epithelium at least 3 mm. proximal to the junction of the tubular esophagus and the stomach. In
advanced achalasia with megaesophagus, the dilated esophagus may have a capacity of 1 to 2 liters, and
simply fasting overnight does not ensure that the esophagus is empty of food and drink consumed the
day before. Life-threatening massive regurgitation and aspiration may occur as the endoscope is being
introduced unless an effort is made to evacuate the esophagus first by means of a nasogastric tube.

12. Which of the following statements about the diagnosis and treatment of esophageal leiomyomas
is/are correct?
A. The majority are diagnosed after they cause dysphagia and chest pain.
B. Biopsy is indicated at the time of esophagoscopy, to rule out carcinoma.
C. Full-thickness elliptical excision of the esophageal wall is the preferred surgical approach.
D. Endoscopic ultrasonography is a reliable means of following leiomyomas conservatively.
E. Recurrence of resected leiomyomas is minimized by wide local excision.
Answer: D

DISCUSSION: Most esophageal leiomyomas are asymptomatic when discovered incidentally on a barium
esophagogram or upper gastrointestinal tract series. When suspected on the basis of its radiographic
appearance, biopsy of the mass should not be performed at the time of esophagoscopy, so that
subsequent extramucosal resection will not be complicated by scarring at the biopsy site. The preferred
surgical approach is submucosal enucleation of the mass, not full-thickness excision. Leiomyomas have a
characteristic hypoechogenic homogeneous appearance on esophageal ultrasonography that allows a
noninvasive diagnosis and means of surveillance. Submucosal enucleation of leiomyomas, without wide
local excision, provides excellent long-term results with virtually no local recurrence rate.

13. Which of the following statements regarding the pathology of esophageal carcinoma is/are correct?
A. Worldwide, adenocarcinoma is the most common esophageal malignancy.
B. Squamous cell carcinoma is most common in the distal esophagus, whereas adenocarcinoma
predominates in the middle third.
C. Patients with Barrett's metaplasia are 40 times more likely than the general population to develop
adenocarcinoma.
D. Metastases from esophageal carcinoma are characteristically localized to regional mediastinal lymph
nodes adjacent to the tumor.
E. Achalasia, radiation esophagitis, caustic esophageal stricture, Barrett's mucosa, and Plummer-Vinson
syndrome are all premalignant esophageal lesions that predispose to the development of squamous cell
carcinoma.
Answer: C

DISCUSSION: Histologically, 95% of esophageal cancers worldwide are squamous cell carcinomas, but the
incidence of adenocarcinoma is increasing dramatically in the United States and Europe. Squamous cell
carcinoma predominates in the upper and middle thirds of the esophagus, whereas adenocarcinoma is
the most frequent distal esophageal cancer. A columnar lined lower esophagus (Barrett's metaplasia) is
associated with an incidence of adenocarcinoma approximately 40 times greater than that of the general
population. Esophageal cancer is a biologically aggressive tumor that characteristically metastasizes
widely to regional and distant lymph nodes as well as to liver and lungs. Recognized premalignant
esophageal lesions include achalasia, radiation esophagitis, caustic stricture, Plummer-Vinson syndrome,
leukoplakia, esophageal diverticula, and Barrett's metaplasia. All but Barrett's metaplasia are associated
with the development of squamous cell carcinoma.

14. Which of the following statements about the surgical treatment of esophageal carcinoma is/are
correct?
A. The finding of severe dysphagia in association with Barrett's mucosa is an indication for an antireflux
operation to prevent subsequent development of carcinoma.
B. Long-term survival is improved by radical en bloc resection of the esophagus with its contained tumor,
adjacent mediastinal tissues, and regional lymph nodes.
C. The morbidity and mortality rates for cervical esophagogastric anastomotic leak are substantially less
than those associated with intrathoracic esophagogastric anastomotic leak.
D. The leading complications of transthoracic esophagectomy and intrathoracic esophagogastric
anastomosis are bleeding and wound infection.
E. Transhiatal esophagectomy without thoracotomy achieves better long-term survival than transthoracic
esophagectomy.
Answer: C

DISCUSSION: Severe dysplasia in Barrett's mucosa is indicative of carcinoma in situ and is an indication
for resectional therapy, not an antireflux operation. In the majority of patients, local tumor invasion or
distant metastases preclude cure when esophageal carcinoma is diagnosed, and attempts to improve
survival with a more radical local operation performed in the face of systemic disease have been
disappointingly futile. A cervical esophagogastric anastomotic leak causes a relatively minor cervical
salivary fistula that heals in 7 to 10 days in 95% of patients. In contrast, an intrathoracic esophagogastric
anastomotic leak results in mediastinitis, which is fatal in 50%. The leading complications of
transthoracic esophagectomy and an intrathoracic esophagogastric anastomosis are respiratory
insufficiency (from combined thoracic and abdominal incisions) and anastomotic leak resulting in
mediastinitis and sepsis. Both complications are minimized by transhiatal esophagectomy without
thoracotomy plus cervical esophagogastric anastomosis. No single operative approach to the treatment
of esophageal cancer has proved superior to others in terms of long-term survival. The biologic behavior
of the tumor (its stage and aggressiveness)—not the number of lymph nodes resected with the tumor—
determines survival.

15. The best management for a 48-hour-old distal esophageal perforation is:
A. Antibiotics and drainage.
B. Division of the esophagus and exclusion of the perforation.
C. Primary repair with buttressing.
D. Resection with cervical esophagostomy, gastrostomy, and jejunostomy.
E. T-tube fistula and drainage.
Answer: C

DISCUSSION: When the esophagus is repaired primarily and covered by well-vascularized autologous
tissue, the rates of fistula and death are significantly less than those observed for patients who receive
simple repair without any protection. Primary repair with buttressing is the first choice for treatment.
Resection is reserved for esophageal perforations with extensive damage to the esophageal wall or with
advanced mediastinal infection and sepsis. Exclusion of the perforated esophagus and T-tube drainage of
a perforation are alternative approaches that cannot be considered for primary treatment. Antibiotics
and drainage as the sole treatment is reserved for a very small, selected population of patients with
well-contained esophageal perforation.

16. A 50-year-old patient develops sudden left lower chest pain and epigastric pain after vomiting. The
patient shows diaphoresis, breath sounds are decreased on the left, and there is abdominal guarding.
The most appropriate diagnostic test is:
A. Aortography.
B. Esophagoscopy.
C. Electrocardiogram.
D. Film of the chest.
E. White blood count.
Answer: D

DISCUSSION: The history of pain after vomiting efforts suggests esophageal rupture. Pain is often
described as excruciating and frequently masquerades as a dissecting aneurysm, perforated ulcer, or
myocardial infarction. Decreased breath sounds suggest the possibility of hydropneumothorax. The
diagnostic procedure is a chest film. More than 90% of patients with an esophageal perforation show
abnormalities suggestive of perforation. The findings are influenced by the interval between perforation
and the examination, by the site of the perforation, and by the integrity of the mediastinal pleura.
Esophagoscopy is not indicated.

17. The following statements about the influence of diet and lifestyle on lower esophageal sphincter
(LES) function are true except one. Identify the incorrect statement.
A. A high-protein diet increases LES pressure.
B. A fat meal results in sustained decrease in LES pressure.
C. Chocolate ingestion causes a decrease in LES pressure.
D. Peppermint produces a transient decrease in LES values.
E. Cigarette smoking produces no significant changes in LES pressures.
Answer: E

DISCUSSION: There is a dramatic decrease in LES pressure following the ingestion of fat. Chocolate has
the same effect on LES resting pressures. Peppermint was shown to produce transient decreases in LES
pressures of 20 to 30 seconds, which occur approximately 10 minutes after ingestion. The ingestion of
carbohydrates produces no significant change in sphincter pressures, but a high-protein meal increases
LES pressure. Cigarette smoking has also been shown to produce significant decreases in LES pressure
that persist throughout the duration of active smoking.

18. When a stricture is present in association with gastroesophageal reflux, each of the following is an
acceptable repair for reflux control except one. Identify the poorest repair.
A. Intrathoracic total fundoplication.
B. Lengthening gastroplasty with total fundoplication.
C. Total fundoplication.
D. Lengthening gastroplasty with partial fundoplication.
E. Partial fundoplication.
Answer: E

DISCUSSION: When a stricture is present, periesophagitis and shortening limit the chances of obtaining a
sufficient length of intra-abdominal esophagus. Even extensive mobilization of the esophagus to the
aortic arch and freeing of the esophagogastric junction does not afford a comfortable 4 to 5 cm. of
esophagus under the diaphragm. A tension-free repair is not possible in such circumstances. Partial
fundoplication at this stage of the disease is followed by a 45% failure rate. Excellent results have been
reported using a total fundoplication following dilatation of the stricture, and intrathoracic
fundoplication provided good results. The lengthening gastroplasty with a partial fundoplication or with
total fundoplication shows satisfactory control of reflux in a majority of patients.

19. When assessing gastroesophageal reflux disease by manometry each of the following statements is
correct except one. Identify the incorrect one.
A. Absent or extremely low LES pressures have predictive value in identifying more severe reflux.
B. Peristaltic dysfunction increases with increasing severity of esophagitis.
C. With established reflux disease the UES is hypertensive.
D. Esophageal functional changes are worst in patients with a circumferential columnar-lined esophagus.
E. Absence of peristalsis may be associated with more severe forms of reflux disease.
Answer: C

DISCUSSION: In reflux disease when LES pressure is below 10 mm. Hg, manometry is too imprecise to
identify a potential for significant reflux. If the pressure is less than 6 mm. Hg, this shows a reasonable
high specificity as compared with abnormal reflux on pH testing. When LES pressure is extremely low or
nonexistent, this identifies a more severe degree of reflux and a poorer prognosis for long-term medical
therapy. In the esophageal body, active reflux esophagitis causes altered function. Failed peristalsis
increases, and the contractions become weaker. Patients with a columnar-lined esophagus have the
worst functional abnormalities. Although distention or acid perfusion in the proximal esophagus can
produce a significant increase in UES resting pressure, there is at present no solid evidence relating UES
resting pressures to active reflux disease.

20. The presence of a nonmalignant mid- or upper esophageal stricture always indicates the presence of:
A. Alkaline reflux esophagitis.
B. Barrett's esophagus.
C. Idiopathic reflux disease.
D. Mediastinal fibrosis.
E. Scleroderma.
Answer: B
DISCUSSION: A stricture at or above the aortic arch is almost certainly situated above an esophagus lined
at its lower end with columnar epithelium. Barrett's esophagus is suggested radiographically when local
esophagitis, ulcer, or stricture is at the limits of a normal-looking segment of esophagus under the
stricture but above a herniated stomach. The columnar-lined esophagus is not always associated with a
high stricture; however Messiaen and Halpert documented strictures in 80% of their Barrett's patients.
These high strictures, when seen with alkaline reflux esophagitis, with idiopathic reflux disease, or in
association with scleroderma, always suggest the presence of a columnar-lined esophagus. Mediastinal
fibrosis is a rare condition that can cause multilevel strictures on the esophagus.

21. Which of the following is most reliable for confirming the occurrence of a significant esophageal
caustic injury?
A. History of the event.
B. Physical examination of the patient.
C. Barium esophagraphy.
D. Endoscopy.
Answer: D

DISCUSSION: In the absence of physical or radiographic evidence of upper airway obstruction or


esophagogastric perforation, the presence of a significant caustic injury can be defined reliably only by
direct visualization at the time of endoscopy. Although the history may shed light on the possibility of a
burn and its severity, all too often the event goes unwitnessed or the type and amount of ingested
substance are not known with certainty. The identification of oropharyngeal burns clearly indicates the
need for endoscopy, but as many as 70% of patients with such lesions escape associated esophageal
injury. Conversely, 10% to 30% of patients with no external evidence of burns have subsequently been
confirmed by esophagoscopy to have sustained damage. In the absence of an identifiable perforation, a
barium esophagogram can rarely be considered unequivocally diagnostic of acute injury, though such an
injury may be suggested when the esophagus appears atonic and dilated, rigid and persistently
narrowed, or excessively irritable. Because of the importance of early confirmation of the presence or
absence of a significant esophageal burn as a guide to formulating appropriate treatment,
esophagoscopy should be performed expeditiously as soon as sufficient time has elapsed to allow gastric
emptying and stabilization of the patient, preferably within the first 12 to 48 hours after ingestion.

22. Indications for surgical reconstruction of the esophagus include which of the following?
A. Continuing requirement for frequent dilation of an extensive esophageal stricture for a minimum of 2
years.
B. Failure or refusal of the patient to comply with a treatment regimen of regular dilation.
C. Development of a fistula between the esophagus and tracheobronchial tree.
D. Iatrogenic perforation of the esophagus during attempted dilation.
Answer: BCD

DISCUSSION: Development of a tracheobronchial fistula almost always necessitates some form of


esophageal reconstruction because of the extensive damage usually associated with it. Failure of the
patient to cooperate effectively condemns to failure any attempt at restoring esophageal patency by
bougienage, because sporadic attempts at dilation do not allow progressive lumen enlargement but,
rather, invite additional injury because of the necessity for repeated instrumentation of a recurrent,
tight stricture. The need for repeated dilation of extensive or multiple strictures over a period
exceeding 6 months should prompt surgical reconstruction of the esophagus, especially in young
children, for whom the psychological and physical hazards are intensified by prolonged treatment. In
such circumstances, a 2-year period of attempted bougienage is excessive. Although iatrogenic
perforation often signals the need for esophageal reconstruction, this misadventure should not be
considered an absolute indication but should be assessed in relation to (1) the extent and complexity of
the stricture, (2) the potential for eventually achieving successful bougienage, and (3) the severity of
complications caused by the secondary injury.

23. First-line therapy for routine peptic duodenal ulcer disease includes:
A. Vagotomy and antrectomy.
B. Upper endoscopy and biopsy to rule out tumor.
C. Evaluation for Helicobacter pylori.
D. Serum gastrin determination.
E. Cream or milk-based “Sippy” diet.
Answer: C

DISCUSSION: Vagotomy and antrectomy is the definitive surgical therapy for peptic ulcer disease but
should be applied only for complications of the disease or after refractory disease has been documented.
Biopsy of routine peptic duodenal ulcer is not indicated to rule out malignancy except in special
circumstances, such as an endoscopic appearance typical of malignancy. H. pylori is found in a large
percentage of peptic ulcer patients, and treatment alters the rate of recurrence if therapy is directed
toward reduction of H. pylori in addition to acid. Measurement of serum gastrin is recommended for
patients with resistant or persistent peptic ulcer disease for patients undergoing surgery for peptic ulcer
disease. The Sippy diet has not been recommended clinically for years. Formerly it was recommended as
a bland diet that would not exacerbate peptic ulcer disease. It is now known that these diets are heavily
calcium-laden and probably exacerbate peptic ulcer disease.

24. Appropriate management of severe vomiting associated with gastric outlet obstruction from peptic
ulcer disease includes all of the following except:
A. Nasogastric suction.
B. Intravenous hydration.
C. Nutritional assessment; upper endoscopy to rule out malignancy.
D. Intravenous H 2 antagonist.
E. Oral antacid therapy.
Answer: E

DISCUSSION: All patients should undergo nasogastric suction, rehydration, and control of acid secretion.
This control of acid secretion requires an H 2 antagonist since oral antacids are often inadequate to
neutralize the large volume of acid often present in the obstructed stomach. Patients with a long history
of obstruction are often nutritionally compromised and need careful nutritional assessment before
operative planning.

25. All of the following are complications of peptic ulcer surgery except:
A. Duodenal stump blowout.
B. Dumping.
C. Diarrhea.
D. Delayed gastric emptying.
E. Steatorrhea.
Answer: E

DISCUSSION: Duodenal stump blowout occurs after Billroth II operations, where back-pressure in the
duodenal stump results in breakdown of this stump closure, leading to abdominal sepsis. Dumping
syndrome and postvagotomy diarrhea are complications of peptic ulcer surgery. They represent two
different syndromes, both of which are predominately related to the vagotomy portion of the operation.
Delayed gastric emptying occurs frequently after peptic ulcer surgery, for a variety of reasons, but it is
most common after elective peptic ulcer surgery for gastric outlet obstruction. Steatorrhea is not
necessarily related to peptic ulcer surgery but is a complication of pancreatic insufficiency.

26. The presentation of Zollinger-Ellison syndrome includes all of the following except:
A. Hyperparathyroidism in patients with multiple endocrine neoplasia type 1 (MEN 1) syndrome.
B. Diarrhea.
C. Migratory rash.
D. Jejunal ulcers.
E. Duodenal ulcers.
Answer: C

DISCUSSION: Zollinger-Ellison syndrome occurs in two settings: sporadically and in association with MEN 1
syndrome. MEN 1 syndrome includes parathyroid adenomas, and the initial presentation is often related
to this parathyroid disease. Diarrhea is a common presentation for Zollinger-Ellison syndrome, since
hyperacidity can result in diarrhea due to the volume of acid secreted or from a steatorrhea-type
diarrhea when the high levels of acids inactivate the pancreatic enzymes. Migratory rash is commonly
associated with glucagonoma but not with Zollinger-Ellison syndrome. Both jejunal and duodenal ulcers
can be found with Zollinger-Ellison syndrome.
27. All are true about the dumping syndrome except:
A. Symptoms can be controlled with a somatostatin analog.
B. Diarrhea is always part of the dumping syndrome.
C. Flushing and tachycardia are common features of the syndrome.
D. Separating solids and liquids in the patient's oral intake alleviates some of the symptoms of the
syndrome.
E. Early postoperative dumping after vagotomy often resolves spontaneously.
Answer: B

DISCUSSION: The somatostatin analog octreotide has been used to control the dumping syndrome and is
currently the only known medical therapy for this disease. Other therapies include dietary measures
such as six small meals a day and separation of solids and liquids. Postvagotomy diarrhea is a secondary
complication of vagotomy and is not strictly associated with the dumping syndrome itself. The vast
majority of patients with dumping syndrome experience spontaneous resolution of their symptoms
without intervention in the postoperative period.

28. In patients with bleeding duodenal ulcers, the endoscopic finding associated with the highest
incidence of rebleeding is:
A. Visible vessel.
B. Cherry-red spot.
C. Clean ulcer bed.
D. Duodenitis.
E. Shallow, 3-mm. ulcer.
Answer: A

DISCUSSION: A visible vessel in an ulcer bed is associated with a 50% chance of rebleeding and, other
than an actively bleeding vessel, is the worst endoscopic prognostic indicator for rebleeding. Cherry-red
spot, adherent clot, and clean small ulcers all are associated with a lower incidence of rebleeding.

29. All of the following are contraindications for highly selective vagotomy except:
A. Intractable duodenal ulcer disease.
B. Peptic ulcer disease causing gastric outlet obstruction.
C. Fundic peptic ulceration.
D. Cigarette chain smoking.
E. Perforated peptic ulcer disease with more than 24 hours' soilage.
Answer: A

DISCUSSION: Intractable peptic ulcer symptoms are a classic indication for highly selective vagotomy.
Patients with gastric outlet obstruction often do poorly with highly selective vagotomy and develop
recurrent ulceration. Highly selective vagotomy is not indicated for gastric ulceration. Heavy chain
smokers often get recurrent peptic ulceration after highly selective vagotomy; therefore, vagotomy and
antrectomy is indicated for them. Patients who experience long periods of perforation before
exploratory laparotomy should receive either patch plus vagotomy or pyloroplasty or patch of the ulcer
alone. Extensive operations, such as highly selective vagotomy, are usually not indicated in this acute
setting.

30. All the following are true of omeprazole except:


A. It is the only drug available that has the potential to achieve pharmacologically induced achlorhydria.
B. It works by blocking the hydrogen-potassium ATPase in the parietal cell.
C. It is parietal cell specific.
D. It has a short half-life (about 90 minutes) when taken orally.
E. It has been associated with gastric neoplasm in a rat model.
Answer: D

DISCUSSION: Omeprazole and drugs in this category are the only drugs that can produce achlorhydria. All
other antiacid drugs reduce acid secretion without producing achlorhydria. Omeprazole inhibits acid at
the final common pathway by blocking the hydrogen-potassium ATPase in parietal cells. It is gastric
parietal cell specific and has a very long half-life, allowing once daily dosing when given orally. When it
was given to rats in pharmacologic doses the gastric mucosa formed carcinoid-type tumors. This problem
has not been identified in humans.

31. All of the following statements about gastrin-releasing peptide (GRP) are true except:
A. In species other than man and dog GRP is commonly referred to as bombesin.
B. GRP serves as a neurotransmitter.
C. GRP inhibits pancreatic secretion when given intravenously.
D. GRP stimulates gastric acid secretion when given intravenously.
E. GRP is released in response to cholinergic stimulation of the parietal cells to stimulate release of
gastrin.
Answer: C

DISCUSSION: Gastrin-releasing peptide and bombesin are homologous peptides of different amino acid
lengths. GRP functions as a neurotransmitter at the cholinergic nerve ending on the parietal cell and
releases gastrin after cholinergic stimulation. It functions to increase gastric acid secretion and also
pharmacologically increases pancreatic secretion.

32. Cholecystokinin (CCK) is believed to function in all of the following processes except:
A. It physiologically delays gastric emptying.
B. It appears to have a role in satiety regulation.
C. It contracts the gallbladder.
D. It stimulates pancreatic secretion.
E. It is important in the control of the anal sphincter.
Answer: E

DISCUSSION: CCK has a physiologic role in the regulation of gastric emptying, eating behavior,
gallbladder contraction, and pancreatic secretion. There is experimental evidence that it may serve as a
neurotransmitter in the function of the lower esophageal sphincter. It probably also has a role in
augmenting the release of insulin after a meal. It has no known role in the function of the anal
sphincter.

33. All of the following measures have been recommended for control of acid secretion in patients with
Zollinger-Ellison syndrome except:
A. Antrectomy.
B. Highly selective vagotomy.
C. Total gastrectomy.
D. Vagotomy and pyloroplasty.
E. Medical therapy with Prilosec (omeprazole).
Answer: A

DISCUSSION: Patients with MEN 1 syndrome or sporadic-metastatic Zollinger-Ellison syndrome should be


palliated with omeprazole to control their acid secretion. Patients who undergo exploration may have a
variety of operations to control their ulcer diathesis, including total gastrectomy or various vagotomy-
type operations. Antrectomy alone is not indicated, since the gastrin that is contributing to the
production of acid is not coming from the antrum but coming from the tumor.

34. All of the following contribute to peptic ulcer disease except:


A. Cigarette smoking.
B. Nonsteroidal anti-inflammatory drugs.
C. Helicobacter pylori.
D. Gastrinoma.
E. Spicy foods.
Answer: E

DISCUSSION: Cigarettes and nonsteroidal anti-inflammatory drugs are common contributors to peptic
ulceration. H. pylori is found in most patients with peptic ulceration, and eradication of this bacterium
decreases the recurrence rate for peptic ulcer disease. Gastrinoma results in much acid secretion and
commonly presents with peptic ulcer disease. Dietary factors such as spicy foods have little or no effect
on postprandial acid secretion and do not contribute to peptic ulceration.
35. Which of the following statements about gastric polyps is/are true?
A. Like their colonic counterparts, gastric epithelial polyps are common tumors.
B. They are analogous to colorectal polyps in natural history.
C. Endoscopy can uniformly predict the histology of a polyp based on location and appearance.
D. In a given patient, multiple polyps are generally of a single histologic type.
E. Gastric adenomatous polyps greater than 2 cm. in diameter should be excised because of the risk of
malignant transformation.
Answer: DE

DISCUSSION: As early as 1895 Hauser reported an association between familial adenomatous polyposis of
the colon and multiple gastric polyps. This early association may have given rise not only to the
confusing nomenclature of gastric polyps but also to the mistaken notion that they are analogous to
colorectal polyps in microscopic appearance and natural history. Unlike colonic polyps, gastric epithelial
polyps are very uncommon tumors (prevalence 0.4% to 0.8%). Their histologic appearance cannot be
predicted on the basis of location in the stomach, although the endoscopic literature is beginning to
define predictive algorithms based on location and ultrasound. Multiple polyps are almost always of a
single histologic type. Gastric adenomatous polyps have long been associated with adenocarcinoma. This
association is directly related to the size of the polyps. Up to 24% of polyps 2 cm. or greater in diameter
are associated with adenocarcinoma. In contrast, only 4% of polyps with a diameter less than 2 cm. are
associated with carcinoma. The risk, if any, of carcinoma in patients with hyperplastic polyps appears to
be associated with the atrophic gastritis that frequently accompanies them rather than with the polyps
themselves.

36. Which of the following statements about gastric leiomyomas is/are true?
A. They are the most common type of gastric tumor of the stomach at autopsy.
B. The leiomyoblastoma cell type reflects malignant transformation of gastric leiomyomas.
C. A conservative surgical approach is indicated for their resection since regional lymphadenectomy has
not been proved reliable even when they turn out to be malignant.
D. Severe hemorrhage may occur from deep ulcerations overlying the intramural tumor.
Answer: ACD

DISCUSSION: Approximately 40% of benign tumors of the stomach are leiomyomas derived from the
smooth muscle of the stomach or its associated blood vessels. Because it is rare for gastric leiomyomas
smaller than 3 cm. in diameter to be symptomatic, considerably fewer than 2% of gastric neoplasms
resected surgically are of smooth muscle origin. Gastric leiomyomas may be smooth or lobulated, but in
time a central ulceration occurs in the mucosal bulge of the tumor in approximately half of submucosal
leiomyomas. Ulceration may be present in smaller tumors but absent in very large tumors. Overlying
central mucosal ulceration, which may penetrate deeply into the tumor, results in hematemesis,
melena, or anemia and draws attention to the tumor. Bleeding from the tumor may be massive and/or
intermittent.
Gastric leiomyomas are not encapsulated, even though on section they appear to be well-circumscribed.
Microscopically, the tumor cells at the margin may intermingle with cells of the surrounding gastric wall.
Along with the presence of occasional large cells with hyperchromatic nuclei, this has led to confusion in
distinguishing benign tumors from malignant ones. Stout described a reasonably distinct variety of
gastric smooth muscle tumor that he called leiomyoblastoma (bizarre smooth muscle tumor). They were
characterized histologically by polyhedral smooth muscle cells with central nuclei and abundant
cytoplasm rather than elongated cells. A clear zone that surrounds the central nucleus may be an
artifact of fixation. Leiomyoblastoma may be benign or malignant. Carney has described a syndrome
characterized by the triad of multiple malignant leiomyoblastoma, pulmonary chondroma, and
functioning extra-adrenal paraganglioma.
The principle of surgical treatment of smooth muscle tumors is local excision with a 2- to 3-cm. margin
of surrounding gastric wall. In view of the difficulty in distinguishing between the benign and malignant
variants, enucleation is not an appropriate method of treatment. Regional lymphadenectomy is not of
proven value, even if malignancy is strongly suspected and is not consistent with the known property of
these tumors to spread by the hematogenous route.

37. The sine qua non of the histologic diagnosis of a gastric pseudolymphoma is:
A. Extragastric extension of the gastric lesion.
B. Nodal involvement beyond the immediate stomach.
C. A germinal center in the gastric lesion.
D. Extension into esophagus and duodenum.
E. Unresponsive to conservative gastric resection.
Answer: C

DISCUSSION: Pseudolymphoma represents approximately 10% of all gastric lymphomas. These are benign
conditions involving mostly the mucosa without evidence of nodal disease and without extragastric
extension. The sine qua non for the diagnosis is a germinal center within the gastric lesion. These are
premalignant lesions but can be cured completely with conservative resection.

38. All of the following statements about surgical management of gastric lymphomas are true except:
A. Stage I gastric lymphomas (small lesions confined to the stomach wall) can be cured completely with
surgical therapy alone.
B. Extensive gastric lymphomas that initially are treated with radiation and/or chemotherapy
occasionally perforate during treatment and require secondary resection.
C. Patients explored with a presumptive diagnosis of gastric lymphoma should undergo an attempt at
curative resection when this is safe and feasible.
D. Without a preoperative diagnosis resection for gastric mass should not be attempted unless lymphoma
can be excluded.
E. Appropriate staging for primary gastric lymphoma includes bone marrow biopsy.
Answer: D

DISCUSSION: Operation alone is adequate treatment for very early-stage lymphoma, although
chemotherapy is commonly added. For more advanced disease, particularly stages III and IV,
preoperative radiation chemotherapy is often indicated, even though some of these patients suffer
perforation during therapy and require emergent resection. Patients who undergo exploration for gastric
mass without a preoperative diagnosis can safely be resected with potential for cure even if the
diagnosis includes gastric lymphoma.

39. Which of the following risk factors have been shown to increase significantly the incidence of
gastrointestinal bleeding from stress gastritis in intensive care unit (ICU) patients?
A. Glucocorticoid administration.
B. Respiratory failure.
C. Coagulopathy.
D. Organ transplantation.
E. Jaundice.
Answer: BC

DISCUSSION: Prophylactic measures such as H 2-receptor antagonists and antacid titration effectively
reduce the incidence of gastrointestinal bleeding; however, prophylaxis against stress gastritis is
expensive and may have adverse effects. Therefore, it should be used selectively in patients with high
risk factors. In a prospective multicenter study in which 10 potential risk factors were evaluated for
stress gastritis bleeding in ICU patients, respiratory failure and coagulopathy are two independent risk
factors for clinically significant bleeding. Therefore, a strong recommendation for prophylaxis of stress
gastritis can be made for ICU patients who have either respiratory failure or coagulopathy.

40. Which of the following measures are effective in preventing stress gastritis bleeding in critically ill
patients?
A. Improving systemic circulation by correcting any shocklike state resulting from blood loss or sepsis.
B. Correcting systemic acid-base abnormality.
C. Maintaining adequate nutrition.
D. Reducing intragastric acidity by either antacid titration or H 2 antagonists.
Answer: ABCD

DISCUSSION: Despite the lack of documentation, a strong impression exists among clinicians and clinical
investigators that the incidence and prevalence of stress gastritis have decreased significantly during the
past decade, perhaps owing to improved general care for critically ill patients. The improvement in
general care of these critically ill patients includes vigorous efforts to correct any shocklike state
secondary to blood loss or sepsis, better ventilatory support, and maintenance of adequate nutrition.
These prophylactic measures enhance the ability of the gastric mucosa to protect itself against acid
injury. In addition, several prospective, randomized studies have shown that antacid titration and/or H
2-receptor antagonists are effective in preventing gastrointestinal bleeding in these patients.

41. Which of the following have been used successfully to treat patients with vascular compression of
the duodenum?
A. Subtotal gastrectomy and Roux-en-Y gastrojejunostomy.
B. Total parenteral nutrition.
C. Division of the ligament of Treitz and duodenal mobilization.
D. Percutaneous endoscopic gastrostomy.
E. Duodenojejunostomy.
Answer: BCE

DISCUSSION: Vascular compression of the duodenum is best treated initially with supportive care. Of
paramount importance is supplying adequate nutrition, since most patients have significant weight loss
with this syndrome. This can best be done with a nasojejunal feeding tube placed past the ligament of
Treitz (and the obstructed area). Gastrostomy alone does not provide unobstructed enteral access.
Parenteral nutrition may be used successfully when enteral access cannot be established. When
operative therapy is needed, duodenojejunostomy has been the most common and successful operation
and is the treatment of choice for adults. In the pediatric population, division of the ligament of Treitz
and duodenal mobilization has also proved successful. Gastrojejunostomy has been used, but with a
lower overall success rate. Distal gastrectomy usually worsens duodenal obstruction by preventing
duodenogastric reflux.

42. Which of the following statements about the anatomic basis for the syndrome of vascular
compression of the duodenum are true?
A. The duodenum is obstructed in its distal third as it crosses over the lumbar vertebral column.
B. Structures crossing beneath the superior mesenteric artery include the duodenum, the uncinate
process of the pancreas, and the left renal vein.
C. Hyperextension of the back allows the angle of origin of the superior mesenteric artery to widen,
lessening the obstruction of the duodenum.
D. Patients are at significant risk for vascular compression of the duodenum if the angle between the
takeoff of the superior mesenteric artery and the aorta is less than 45 degrees.
E. Arteriographic studies show a typical area of extrinsic compression and narrowing of the arterial
lumen due to duodenal pressure.
Answer: AB

DISCUSSION: The superior mesenteric artery originates behind the neck of the pancreas at the level of
the first lumbar vertebra. It arises from the aorta at an acute angle, usually about 37 degrees in normal
patients, through which passes the left renal vein, the uncinate process of the pancreas, and the distal
third of the duodenum. The duodenum crosses the lumbar spine from right to left and passes upward. It
is at this point of passage of the duodenum upward and over the spine that the obstruction occurs.
Arteriographic studies show that the aortomesenteric angle in patients with the syndrome is only about 8
degrees. There is no narrowing of the superior mesenteric artery or disturbance of arterial flow, but the
area of duodenal obstruction corresponds to the compression of the bowel by the artery. The duodenal
compression may often be relieved by assuming the knee-chest, the left lateral, or even the prone
position. Increasing lumbar lordosis, as with hyperextension of the back, exacerbates the problem.

43. Numerous epidemiologic associations have been made between (1) environmental and dietary factors
and (2) the incidence of gastric cancer, including all except:
A. Dietary nitrites.
B. Dietary salt.
C. Helicobacter pylori infection.
D. Dietary ascorbic acid.
Answer: D

DISCUSSION: Numerous epidemiologic studies support the role of certain foods in the development of
gastric cancer. Salt, which can act as a gastric irritant, and nitrates and nitrites, which can be converted
to the active carcinogens N-nitrosamines, are implicated in the development of gastric cancer. H. pylori
infection is associated with atrophic gastritis, a known precursor to gastric cancer. Important studies of
large populations indicate that the majority of patients with gastric cancer are H. pylori positive. The
bacteria produce toxins such as ammonia and acetaldehyde, which could lead to chronic inflammation
and epithelial damage. Dietary ascorbic acid has been associated with overall improvements in diet and
is not associated with the development of gastric cancer.

44. All of the following benign conditions are associated with increased rates of gastric cancer except:
A. Pernicious anemia.
B. Multiple endocrine neoplasia type I (MEN 1).
C. Adenomatous polyps.
D. Chronic atrophic gastritis.
Answer: B

DISCUSSION: Adenomatous polyps are unusual but carry the distinct potential for malignancy. They occur
most often between the fifth and seventh decades of life. The adenocarcinoma sequence in gastric
polyps is thought to be analogous to that of colonic polyps. An adenomatous polyp is a marker for
increased risk of carcinoma in the remaining stomach. Both pernicious anemia and chronic atrophic
gastritis are associated with gastric cancer. Many of these patients develop chronic achlorhydria, a
condition also associated with an increased risk of cancer. Neither multiple MEN 1 nor MEN 2, is
associated with gastric cancer.

45. Which of the following statements concerning the pathology of gastric cancer is true?
A. Distal gastric cancers are becoming more common.
B. Intestinal-type gastric tumors resemble colon carcinomas and have a better prognosis than diffuse
type.
C. Early gastric cancers are confined to the mucosa and are lymph node negative.
D. Broders' histologic grading system correlates well with survival: patients with grade IV tumors have 5-
year survival rates around 65%.
Answer: B

DISCUSSION: Distal gastric cancers are decreasing in incidence in several populations. Lesions of the
gastroesophageal junction and cardia have increased in incidence over the past two decades. Early
gastric cancers are confined to the mucosa and submucosa of the stomach. Six to 10% of these early
lesions are lymph-node positive. The survival rates from early gastric cancer is related to node positivity,
just as in advanced gastric cancer. Broders' histologic grades do correlate well with survival. Grade I and
IV tumors are associated with a 65% and 11% 5-year survival, respectively. The Lauren classification
system is divided into intestinal and diffuse-type tumors. The intestinal-type tumor is more analogous to
colon carcinoma and has a better prognosis than the diffuse type.

46. An 80% distal gastrectomy is performed for a 6-cm. antral cancer with extension to the muscularis
propria and three positive lymph nodes less than 3 cm. from the tumor. The stage of this tumor was:
A. Stage I.
B. Stage II.
C. Stage III A.
D. Stage III B.
Answer: B

DISCUSSION: The American Joint Committee on Cancer Staging system depends on primary tumor, lymph
node involvement, and distant metastasis. The tumor described is a T2N1M0 tumor, which categorizes it
as stage II.

47. Which of the following statements about the surgical treatment of gastric cancer is false?
A. Patients with tumors of the middle and proximal thirds should undergo total gastrectomy.
B. Adenocarcinoma of the cardia-gastroesophageal junction may require reconstruction in the abdomen,
chest, or neck.
C. Palliative resection yields better results than palliative bypass.
D. Japanese patients who undergo gastric resection are, on average, 10 years younger and much leaner
than their Western counterparts.
Answer: A

DISCUSSION: Depending on the size and extent of the tumor, cancers of the gastroesophageal junction
may extend proximally into the esophagus for a varying distance. Reconstruction may be required in the
abdomen, chest, or neck, depending on extension and whether the operation is to be palliative or
curative. Palliative resection yields better results than palliative bypass, which is unreliable for relieving
obstruction. Japanese patients typically are younger and thinner than their Western counterparts. In
addition, they have a higher prevalence (up to 50%) of early gastric cancer. Depending on the size and
particular location of the tumor, patients with small middle-third tumors or small lesions of the cardia
may undergo subtotal proximal gastrectomy and reconstruction with a gastric tube. If 6-cm. margins can
be obtained on either side of the lesion, total gastrectomy is unnecessary and may be associated with a
higher risk of morbidity.

48. Which of the following measures of obesity correlates best with mortality?
A. The 1983 Metropolitan Life Insurance Company tables for ideal body weight.
B. Hydroimmersion measurements of body fat composition.
C. Body mass index (BMI).
D. Skinfold thickness.
E. Waist to hip ratios (WHR).
Answer: C

DISCUSSION: The measurement of obesity is still an inexact science. The Metropolitan Life Insurance
tables, although widely used, do not distinguish between lean muscle mass and fat. Accordingly, the BMI
(weight in kg./height in meters) 2, was developed to place greater emphasis on “fatness.” The measure
correlates linearly with mortality tables. Hydroimmersion data are still too sparse to relate to outcome
tables. Skinfold thickness and waist-hip ratios have not been shown to have the accuracy or relevance of
the BMI.

49. The most effective therapy for morbid obesity, in terms of weight control, is:
A. Intensive dieting with behavior modification.
B. A multidrug protocol with fenfluramine, phenylpropanolamine, and mazindol.
C. A gastric bypass with a 40-ml. pouch, a 10- to 20-cm. Roux-en-Y gastroenterostomy.
D. A gastric bypass with a 15-ml. pouch, a 40- to 60-cm. Roux-en-Y gastroenterostomy.
E. Daily exercise with strong emphasis on utilizing all four limbs.
Answer: D

DISCUSSION: Although the various nonsurgical measures listed in the question have proved effective for
obese persons, they work only rarely for those who are morbidly obese. None have proved as effective as
gastric bypass with a 15-ml. pouch and a 40- to 60-cm. Roux-en-Y gastroenterostomy.

50. Which of the following statements about intestinal bypass is/are correct?
A. The operation produced weight loss similar to that of the gastric bypass.
B. The operation produced severe metabolic disturbances, including hypocalcemia, increased bile salts
and glycine synthesis.
C. Bacterial overgrowth in the bypassed segment led to liver failure.
D. The operation demonstrated that an adult human could survive with 40 to 50 cm. of small intestine.
Answer: ABCD

DISCUSSION: Unfortunately, all of the answers are true. Even though the intestinal bypass proved initially
to be an effective procedure to induce weight loss, the side effects proved to be so severe that almost
all of the operations had to be reversed or revised to gastric bypass to avert death from liver failure or
severe illnesses due to malnutrition.

51. Which of the following is/are contraindications to gastric bypass surgery?


A. Diabetes mellitus.
B. Hypertension.
C. Pickwickian syndrome.
D. Failure to agree to long-term follow-up.
E. Sleep apnea.
Answer: D

DISCUSSION: The gastric bypass represents the best known therapy for diabetes mellitus, hypertension,
the Pickwickian syndrome, and sleep apnea. In fact, no other therapy provides such complete control of
hyperglycemia and hyperinsulinemia, reversal of hypertension, and total correction of the Pickwickian
syndrome and most cases of sleep apnea. The only contraindication to bariatric surgery listed is failure
to agree to long-term follow-up. The gastric bypass represents controlled malnutrition and, therefore,
vitamin therapy is especially important. If patients are not followed closely, vitamin deficiencies,
especially of B 6 and B 12, can develop with serious consequences including a Korsakoff-Wernicke
syndrome.

52. A 34-year-old morbidly obese diabetic woman underwent a gastric bypass about 12 hours ago. The
operation was technically difficult but finally went well. You are called because she now has a
temperature of 99.2‫ ؛‬F, pulse of 134, and some pain in her incision and her back. She looks well; the
incision is clean; and her examination is otherwise negative. A bolus of 500 ml. of dextrose/lactated
Ringer's did not change her vital signs, except that her pulse rose to 140 without an increase in urine
output. Your next step should be:
A. Another bolus of crystalloids.
B. Posterioanterior and lateral chest films.
C. Obtain white cell count, differential count, and electrolyte values.
D. Call the operating room and warn them that you need to re-explore for a leak.
E. Increase her pain medication.
Answer: D

DISCUSSION: Morbidly obese patients are malnourished and brittle and have little resistance. Serious life-
threatening infections may soil the peritoneal cavity without producing any sign except a persistently
high pulse rate. The usual tests listed in A, B, and C, should be done, but the most likely explanation for
a continued high pulse is soiling in the area of the surgery due to either a leak or contamination and
development of sepsis. Because of the unreliability of clinical evaluation, the indications for re-
exploration are very liberal, and this approach has saved a number of lives. The risk of such an
exploration is small, whereas failure to contain the infection with lavage and drainage may be followed
by a surprisingly rapid death.

53. Metabolic complications of subtotal gastrectomy with Billroth I or Billroth II reconstruction include:
A. Hypothyroidism.
B. Anemia.
C. Reactive hypoglycemia.
D. Dumping syndrome.
E. Metabolic bone disease.
Answer: BCDE

DISCUSSION: Anemia develops in as many as 30% of patients within 15 years of surgery. The cause is
multifactorial and includes malabsorption of iron, folate, and vitamin B 12. A metabolic bone disease
occurs in as many as 33% of patients, is similar to osteomalacia, and is probably a result of
malabsorption of calcium and vitamin D. Reactive hypoglycemia occurs with rapid gastric emptying,
resulting in increased glucose absorption immediately after a meal. Initially there is hyperglycemia,
leading to hyperinsulinemia and subsequent rapid glucose clearance and symptomatic hypoglycemia.
Dumping syndrome varies from very mild symptoms to significantly disabling ones. The severe syndrome
occurs in fewer than 5% of patients. Small, frequent, dry meals of low osmolality reduces symptoms, and
somatostatin analog has been of some clinical use.

54. Which of the following statement(s) concerning the surgical options for an anti-reflux operation
is/are true?

a. A patient with normal esophageal length and esophageal body motility is best served by laparoscopic
Nissen fundoplication
b. A patient with a low peristaltic amplitude of the distal third of the esophagus is a candidate for an
open Nissen fundoplication
c. A Collis gastroplasty is an additional procedure that can be added in patients with extensive
esophageal shortening
d. End-stage reflux disease such as an undilatable stricture or Barrett’s esophagus with high grade
dysplagia is best managed by a colon interposition
Answer: a, c, d
Patients with normal esophageal length and normal esophageal body motility are best served by a
transabdominal Nissen fundoplication. This is now normally done via the laparoscopic route. If the
patient is very obese or requires concomitant surgery on the lung or esophageal body, the transthoracic
approach is preferable. The presence of a motility disorder alters the operative strategy. If the
peristaltic amplitude is low (20 mm Hg) in the distal third of the esophagus, a Nissen fundoplication
would create too much resistance and lead to dysphagia. In this situation the Belsey fundoplication is a
better choice. Moreover, it allows the surgeon to mobilize the esophagus to a much greater extent than
is possible through the abdomen. In addition to extensive mobilization, a Collis gastroplasty can be
created to produce an extra 5 cm of “neo-esophagus” around which a Belsey procedure can be added.
End-stage reflux disease, for example, when there is an undilatable stricture or after previous
unsuccessful anti-reflux operations or when Barrett’s esophagus leads to high grade dysplagia, is best
served by esophageal replacement. The most durable substitute is the colon, and the functional results
are especially good if the vagus nerves are intact.

55. Factors associated with the development of complications of gastroesophageal reflux disease
include:

a. The presence of a defective lower esophageal sphincter


b. Inadequate esophageal clearance
c. The presence of a hiatal hernia
d. The presence of an alkaline component of the reflux material
Answer: a, b, c, d

The status of the lower esophageal sphincter (LES) has emerged as a significant factor in several long-
term studies of gastroesophageal reflux disease, and serves as a predictor of a poor response to medical
treatment. Barrett’s esophagus is almost always associated with a mechanically defective sphincter. Any
defects in esophageal clearance which prolongs the contact time between the refluxate and the mucosa
is likely to lead to increased esophageal injury. The presence of a hiatal hernia is also associated with
more complications of gastroesophageal reflux disease. Finally, the composition of the reflux material
also has an effect on the development of complications. In a clinical situation, complications of
gastroesophageal reflux disease are more common when there is an alkaline component to the refluxate.
In Barrett’s esophagus, the development of complications such as stricture and ulceration is strongly
associated with increased alkaline exposure.

56. Fundamental to understanding disorders of esophageal function is the measurement of the


contractility of the esophageal body and sphincters. Which of the following statement(s) is/are true
concerning esophageal manometry in the investigation of benign esophageal disease?

a. A defective sphincter is predictive of poor long-term response to medical therapy, but a good
response to surgery
b. Esophageal manometry can determine the resting pressure and the overall length of the sphincter but
not its abdominal length
c. The LES pressure normally drops to gastric baseline immediately after a swallow before the peristaltic
wave reaches the lower esophagus
d. A Vector Volume below the fifth percentile of normal is the most sensitive measure of mechanical
deficiency of the LES
e. There is no correlation between defects in LES with the severity of gastroesophageal reflux disease
Answer: a, c, d

Esophageal manometry is an investigative tool in which a catheter containing pressure sensors is inserted
into the esophagus and used to measure pressures in esophageal body and sphincters at rest and in
response to swallowing. It is indicated in a number of clinical situations including nonobstructive
dysphasia, noncardiac chest pain, and the assessment of gastroesophageal reflux disease.
The indications for manometry in patients with suspected gastroesophageal reflux are chiefly to assess
the status of the LES and to identify a motility disorder of the body. A defective sphincter is predictive
of a poor long-term response to medical therapy, but a good response to surgery. The presence of a
motility defect profoundly alters the operative strategy in patients with GERD and should always be
excluded by manometry prior to operative therapy. In assessment of the LES, three components are
measured: the resting pressure, the overall length of the sphincter, and the abdominal length. A defect
in the values for each of these components are determined when the lower limits of normal (fifth
percentile) are determined. A defect in one or even two components of the LES may be compensated for
by good esophageal body function, but when all three components are defective, excessive esophageal
acid exposure is inevitable. All the pressures measured along the length of the sphincter and around its
circumference during the pull-through may be treated as vectors having both magnitude and direction
and hence integrated into a three-dimensional image, the volume (Vector Volume) of which is a measure
of LES resistance. A Vector Volume below the fifth percentile of normal is the most sensitive measure of
mechanical deficiency of the LES. The prevalence of a defective LES increases with increasing severity of
GERD, being the lowest in patients without evidence of endoscopic injury and highest in patients with
stricture or Barrett’s esophagus.

57. Which of the following statement(s) is/are true concerning the diagnosis and management of the
patient whose barium esophogram is shown in Figure 18-29?

a. The condition is due to neuronal generation of the myenteric plexus in the lower esophageal sphincter
b. The patient will report symptoms of vomiting of sour or bitter material
c. Despite the impressive radiologic picture, passage of the endoscope through the area of narrowing will
likely be possible
d. Manometry and 24 hour pH monitoring should be performed for confirmation of the diagnosis
Answer: c

The x-ray demonstrates moderately advanced achalasia with a dilated esophagus with a narrowed
tapering “bird’s beak” appearance of the distal esophagus. Achalasia is the best known primary motility
disorder of the esophagus. It is characterized by failure of the esophageal body peristalsis and
incomplete relaxation of the LES. It is generally thought to be caused by neuronal degeneration of the
myenteric plexus of the esophageal wall, causing aperistalsis, and to loss of activity of the inhibitory
neurons in the LES leading to incomplete relaxation. Patients with achalasia all have dysphagia, and
most have regurgitation. Careful questioning is needed to distinguish the regurgitation from vomiting.
Generally, it occurs during or at the end of a meal, and the material tastes bland rather than sour or
bitter. Patients often have to leave the table to regurgitate, and are usually slow eaters.
Endoscopy frequently reveals residual liquid or food in the esophagus. Unlike a stricture, the narrowing
of the lower end permits the passage of the endoscope, usually with a characteristic “popping”
sensation. In every patient with presumed achalasia, it is very important to view the cardia from below
with the endoscope retroflexed, as a small infiltrating gastroesophageal tumor may otherwise be missed.
Manometry is required to establish the diagnosis of achalasia. The classic features on stationary
manometry are: 1) Elevated LES pressure; 2) Incomplete LES relaxation; 3) Absence of esophageal body
peristalsis; and 4) Positive intraesophageal body pressure. Although reports concerning the use of 24 hr
pH monitoring appear in the literature, excessive acid exposure is rare.

58. Which of the following statement(s) is/are true concerning other tests available for investigation of
esophageal disease?

a. A 24 hour pH monitoring is currently the principal method in making the diagnosis of gastroesophageal
reflux disease (GERD)
b. Acid reflux episodes are defined as periods when the esophageal pH is less than 2
c. Twenty-four pH monitoring is only useful in the detection of acid reflux disease
d. The Bernstein test continues to be an important tool in the diagnosis of acid reflux disease
e. Delayed gastric emptying may be an important etiologic factor in patients with GERD
Answer: a, e

The development of 24 hr pH monitoring was a major advance in unraveling the pathophysiology of


GERD. It is now the principal method to make the diagnosis of GERD and has effectively replaced all
other methods of measuring esophageal acid exposure. It is indicated in any patient with symptoms
suggesting GERD, unless the symptoms are trivial, or permanently abolished by a short course of acid
suppression therapy. Reflux episodes are defined as periods when the esophageal pH is less than pH 4.
Normal (physiologic) reflux occurs in the form of short rapidly cleared postprandial episodes. A few
episodes of long duration are more injurious than many brief episodes, even though total acid exposure
time may be similar. In addition to the measurement of acid exposure, pH monitoring can also be used
to detect excessive alkaline exposure (pH > 7) in the esophagus. The Bernstein test, in which
hydrochloric acid is dripped into the esophagus via a nasogastric tube, is sometimes used to determine if
a patient’s symptoms are reproduced by acidic exposure. It is basically a measure of esophageal mucosal
sensitivity. It has been largely superseded by the use of 24 hr pH monitoring. Gastric emptying is
affected by the composition and consistency of the ingested meal. Delayed gastric emptying may be an
important etiologic factor in patients with GERD and a normal LES.

59. The results for anti-reflux surgery are generally good, however, patients who have failed anti-reflux
procedures constitute a particularly challenging group. Which of the following statement(s) is/are true
concerning failed anti-reflux repairs?

a. A Slipped Nissen is usually the result of an operative technical mistake


b. Disruption of a fundoplication is more prone to occur with a Nissen fundoplication because of the use
of the gastric wall in the repair
c. Postoperative dysphagia in a patient with normal preoperative motility is usually due to a secondary
motility disorder
d. Colonic replacement, although technically challenging, usually has superior long-term results when
compared to esophageal replacement with the stomach
Answer: a, d

When patients are correctly selected and the operation performed with conformity with the basic
surgical principles, long-term relief of symptoms is achieved by more than 90% of patients. A number of
patterns of failure, however, can occur. The so-called Slipped Nissen may develop when the upper
stomach rides up through the fundoplication, and causes both dysphagia and heartburn. It is more likely
that the condition was created at the time of surgery because the surgeon did not mobilize the fundus,
or because unrecognized esophageal shortening led to inadequate mobilization of the gastroesophageal
junction, causing the surgeon to wrap the stomach around the upper stomach rather than the lower
esophagus. Creating too tight a fundoplication leads to immediate postoperative dysphagia. Manometry
shows a high pressure nonrelaxing sphincter which may be difficult to distinguish from achalasia. Such
patients highlight the importance of manometry in all patients before proceeding with anti-reflux
surgery. In a patient with normal preoperative motility, the cause is usually a fault in technique, and can
be prevented by constructing the fundoplication over a 60 F Bougie. Disruption of the fundoplication that
manifests clinically and physiologically by recurrent reflux can be caused by inadequate suture
technique, unrecognized esophageal shortening leading to tension on the wrap, or poor choice of
operation. All partial fundoplications, such as the Toupet procedure are more prone to disruption than a
Nissen. This is because the integrity of the repair depends on sutures to the esophageal wall and not the
stomach, and because all these repairs require much more abdominal length of esophagus than a Nissen,
thus placing the repair under tension. Esophagectomy and esophageal replacement are occasionally
indicated in the treatment of advanced GERD. The indications for esophagectomy are Barrett’s
esophagus with high grade dysplasia and what is generally described “burned out esophagus” which
includes failure of a third anti-reflux operation, a severe coexistent motility disorder, or the presence of
an undilatable stricture. Either colon or stomach may be used to replace the esophagus. Colonic
replacement is more difficult, requiring three anastomoses rather than one, but it has superior
functional long-term results.

60. A number of diagnostic modalities exist for investigation of structural abnormalities of the
esophagus. Which of the following statement(s) is/are true concerning the use of these investigative
studies?

a. Endoscopy should be the first investigation in any patient with foregut symptoms
b. Barrett’s esophagus is suggested when the squamo-columnar junction is more than 2 cm above the
gastroesophageal junction on endoscopic examination
c. There are three areas of esophageal narrowing which can be noted on both barium esophogram and
endoscopy
d. The CT appearance of the esophagus is normally a flattened, hollow structure with a thin wall
Answer: b, c, d

Endoscopy is generally the first investigation in patients with foregut symptoms. The exception is when
the patient’s chief complaint is dysphagia, when a “road map” should first be obtained by a barium
swallow. The locations of the esophageal landmarks are measured endoscopically from the incisor teeth.
Three landmarks are measured in the region of the cardia: the level of the crura, the level of the
anatomic gastroesophageal junction, and the level of the squamo-columnar junction (Z line). A hiatal
hernia is present when the gastroesophageal junction is more than 2 cm above the crura. Barrett’s
esophagus is suggested when the squamo-columnar junction is more than 2 cm above the
gastroesophageal junction but may be diagnosed if any specialized epithelium is identified above the
gastroesophageal junction histologically, regardless of measured length of the columnar segment. Three
areas of esophageal narrowing are frequently noted on both barium esophogram and endoscopy. The
first narrowing is at the site of the cricopharyngeus muscle. The left mainstem bronchus and aortic arch
caused narrowing of the middle third of the esophagus. The most distal narrowing of the esophagus is at
the diaphragmatic hiatus and is caused by the lower esophageal sphincter mechanism. These normal
points of narrowing tend to retard swallowed foreign objects. Also, corrosive liquid ingestion results in
prominent mucosal injury at these sites as the liquid is slowed at passage. CT scan of the esophagus is
important in delineating the relationship of esophageal lesions to adjacent structures, especially the
trachea, left main bronchus and aorta. The esophagus normally appears as a flattened hollow structure
with a thin wall. A more circular cross-sectional appearance with a fluid level is evidence of distal
obstruction.

61. Which of the following patient scenarios would be best managed with anti-reflux surgery?

a. A patient with heartburn but normal 24 hour pH monitoring and an intact lower esophageal sphincter
b. A patient with primarily respiratory manifestations of gastroesophageal reflux
c. A patient with increased acid exposure and a mechanically defective sphincter who responds well to
medical therapy but requires continued long-term medication for continued relief
d. A patient with gastroesophageal reflux but excessive complaints of epigastric pain, nausea, vomiting,
and loss of appetite
Answer: b, c

The first requirement for consideration of anti-reflux surgery is the objective demonstration of the
presence of GERD by 24-hour pH monitoring. Secondly, the patient must have either symptoms or
complications of the disease. Thirdly, the disease should be caused by defect appropriate to surgical
therapy, i.e., a mechanically defective sphincter. Some patients with increased acid exposure and a
mechanically defective sphincter, and who have no complications of the disease respond well to medical
therapy, but they require long-term medication for continued relief. These patients should be given the
option of surgery as a cost effective alternative.
Atypical symptoms of reflux such as respiratory manifestations often respond well to anti-reflux surgery.
When respiratory symptoms are combined with typical symptoms such as heartburn and regurgitation,
the results of anti-reflux surgery are generally good.
Complaints of epigastric pain, nausea, vomiting, and loss of appetite may be due to excessive
duodenogastric reflux which occurs in about 11% of patients with gastroesophageal reflux disease. This
problem is usually, but not invariably, confined to patients who have previous upper gastrointestinal
surgery. The coexistence of these gastric symptoms in a patient who also has typical symptoms of GERD
should prompt a thorough evaluation of the stomach using a bile probe, 24 hour pH monitoring or
radionucleotide scanning. In such patients, the correction of only the incompetent cardia can result in a
disgruntled individual who continues to complain of nausea and epigastric pain on eating.

62. Which of the following statement(s) concerning pharyngoesophageal disorders is/are true?

a. In neuromuscular diseases, dysphagia is often worse for liquids than for solids
b. Cricomyotomy may be indicated for a wide variety of neuromuscular disorders involving the
pharyngoesophageal phase of swallowing
c. Excision of a Zenker’s diverticulum is indicated to prevent malignant change in the sac
d. Complications of all operations on the cervical esophagus include hematoma formation and recurrent
nerve paralysis
Answer: a, b, d

Disorders of the pharyngoesophageal phase of swallowing result from a discoordination of the


neuromuscular events involved in chewing, initiation of swallowing, and propulsion of the material from
the oropharynx to the cervical esophagus. The commonest causes of pharyngoesophageal dysphagia are
neuromuscular diseases. The most important are cerebrovascular disease, myasthenia gravis, Parkinson’s
disease, multiple sclerosis and muscular diseases such as myotonic dystrophy and polymyositis. In
neuromuscular diseases, dysphagia is often worse for liquids than for solids. Choking, repetitive
pneumonia, nasal regurgitation and hoarseness are also prominent features. The surgeon’s role in the
treatment of cricopharyngeal disorders is to reduce outflow resistance by performing a cricomyotomy.
Initially this was recommended only for patients with demonstrable failure of the upper esophageal
sphincter relaxation. More recently, a number of reports indicate a wide variety of neuromuscular
diseases that may be improved by cricomyotomy. The surgical options in Zenker’s diverticulum are
either excision or suspension. Excision is sometimes recommended on the grounds that malignant change
in the sac is prevented, but there is no evidence that excision carries any greater protective role than
suspension, which effectively prevents stagnation of food material, thus removing the presumed cause of
malignant change. Suspension also removes the risk of contamination of the operative site, the risk of
subsequent breakdown of the closure site with fistula formation, and the risk of narrowing of the
esophagus. In either case, recurrence is likely if cricomyotomy is not performed, because the underlying
defect which predisposes to the diverticulum persists. All operations on the cervical esophagus carry the
risk of hematoma formation and recurrent nerve paralysis. The venous pumping action of the lung can
cause the development of a large hematoma in the mediastinum postoperatively, therefore meticulous
hemostasis is critical for the performance of this operation.

63. Barrett’s esophagus is a complication of gastroesophageal reflux disease. Which of the following
statement(s) is/are true concerning this condition?

a. The histologic hallmark is the presence of “specialized” columnar epithelium regardless of how far it
extends into the esophagus
b. Barrett’s epithelium will frequently regress with medical therapy or anti-reflux surgery
c. High grade dysplasia will frequently be associated with foci of invasive carcinoma
d. Patients with adenocarcinoma arising in Barrett’s esophagus have a high incidence of p53 gene
mutations
Answer: a, c, d

Barrett’s esophagus is now recognized as a complication of advanced gastroesophageal reflux disease.


The histologic hallmark of Barrett’s esophagus is the presence of “specialized” columnar epithelium,
which shows features of intestinal metaplasia, easily recognized by the presence of goblet cells. The
presence of specialized epithelium is now regarded as the pathonomonic feature of Barrett’s esophagus
regardless of how high it extends into the esophagus. Barrett’s esophagus may exist on its own, or may
be itself associated with ulceration, stricture, and malignant change. Once Barrett’s epithelium is
present, medical therapy or anti-reflux surgery rarely causes it to regress. Unless it is actually ablated,
for example with laser therapy, it persists. The most significant feature of Barrett’s esophagus is its
malignant potential. The metaplastic epithelium usually undergoes dysplastic change prior to becoming
frankly neoplastic. High grade dysplagia is synonymous with carcinoma in situ, and if the esophagus is
removed for such a condition, up to 50% will demonstrate foci of invasive carcinoma.
In the past, the pathophysiology of Barrett’s esophagus was associated with alkaline reflux on
esophageal pH monitoring. However, more recently using a bile sensor for monitoring bilirubin, this
condition is frequently associated with excessive bile in the esophagus. Repetitive injury from noxious
gastric juice can lead during the repair process to mutations in the p53 gene. Patients with
adenocarcinoma arising in Barrett’s esophagus have a high incidence of p53 mutations.

64. Which of the following statement(s) is/are true concerning the blood supply and lymphatic drainage
of the esophagus?

a. The thoracic esophagus receives no direct branches from the aorta therefore allowing the technique
of transhiatal (blunt) esophagectomy
b. Bleeding esophageal varices are most prominent in the mid-esophagus
c. Lymphatic drainage of the lower third of the esophagus goes entirely to the abdominal lymphatic
system
d. Nodal involvement in esophageal cancer is quite common even if the tumor is limited to the level of
the submucosa
Answer: d

The blood supply and venous drainage of the esophagus are largely segmental. The inferior thyroid
artery provides the main blood supply to the cervical portion of the esophagus. The thoracic portion of
the esophagus receives its blood supply from two sources; branches from two or three bronchial arteries
provide the proximal arterial supply and branches directly from the aorta supply the more distal thoracic
esophagus. Intrathoracic mobilization of the esophagus during performance of anti-reflux procedures
often require ligation of these branches. The venous plexus in the submucosa collects capillary blood and
delivers it into a periesophageal venous plexus. The left gastric vein or coronary vein provides the
principal collateral in portal hypertension when esophageal varices develop. The submucosal veins
become much more superficial in the most distal esophagus, 1–2 cm above the gastroesophageal
junction, and are consequently the most common site of bleeding in portal hypertension.
The lymphatics of the esophagus form a rich submucosal network draining into regional lymph nodes in
the periesophageal connective tissue. There is thus little barrier to longitudinal spread of cancer in the
esophagus. Lymphatic drainage from the upper two-thirds of the esophagus is usually cephalad, but
drainage from the lower one-third is in both directions. Although lymphatic metastasis in the esophagus
generally involve the regional lymph nodes in proximity, nodal involvement may occur several
centimeters away from the primary lesion because of the rich intramural lymphatic anastomotic
channels. When a carcinoma is limited to the mucosa, the incidence of lymphatic metastases is low, but
once into the submucosa, the incidence rises to 60%.

65. Which of the following statement(s) is/are true concerning the process of swallowing and esophageal
transit of food?

a. Injury to the recurrent laryngeal nerves can cause motility problems of the cervical esophagus and
resulting aspiration
b. Esophageal reflux does not lead to impaired esophageal motility
c. Relaxation of the LES is mediated via inhibitory neurons
d. The overall length of the LES is the only factor influencing the pressure gradient of the sphincter
e. A mechanically defective sphincter is always associated with increased esophageal acid exposure
Answer: a, c

The cricopharyngeus muscle is a continuation of the inferior constrictor of the pharynx and receives its
innervation via both the right and left recurrent laryngeal nerves. Although much attention is given to
vocal cord dysfunction that accompanies recurrent laryngeal nerve damage, it is clear that
cricopharyngeal sphincter dysfunction and motility problems of the cervical esophagus can occur with
injury to these nerves. Serious aspiration following recurrent nerve injury is caused not only by the
cricopharyngeal dysfunction, but also by additional morbidity incurred because of the inability to close
the glottis during swallowing and loss of the protection afforded by effective coughing. Clinically,
peristaltic defects of the esophageal body fall into one of to broad categories. One category is
characterized by a defect in organization of peristaltic waves, and is primarily a neural phenomenon.
The other notable defect is reduction of the power (amplitude) of peristalsis and is usually due to muscle
damage secondary to severe reflux or replacement with fibrous tissue as happens in scleroderma and
other connective tissue diseases or with severe reflux. The LES provides a pressure barrier between the
esophagus and stomach. The sphincter normally remains actively closed to prevent reflux of gastric
contents into the esophagus. Relaxation of the LES is mediated by inhibitory neurons. It occurs either to
allow entry of food, or to allow exit of air during belching. The ability of the LES to remain closed in the
face of a pressure gradient tending to promote reflux of gastric contents from the positive pressure
environment of the stomach into the negative pressure environment of the chest depends on several
features. The most significant is the resting pressure. However, of equal importance is the ability of the
LES to respond to variations in intra-abdominal pressure associated with daily activities. Such elevations
would normally be transmitted to the sphincter, causing it to collapse and remain closed, provided
sufficient length of the sphincter remains exposed to the abdominal pressure and the compressive effect
of the crura. The abdominal length is often reduced in hiatal herniation, because of attenuation of the
pharyngoesophageal membrane. The overall length of the LES is also an important determinant of
competence, much as the total resistance of a series of resisters in a circuit is the sum of the individual
resistances. A mechanically-defective sphincter, however, is not always associated with increased
esophageal acid exposure because it may be compensated by the clearance function of the esophageal
body. The role of the esophageal body in limiting acid reflux is related to its ability to clear the
esophagus of acid. This clearance has two components: volume clearance which requires peristalsis, and
chemical clearance which requires saliva.

66. Which of the following statement(s) is/are true concerning the management of this patient?

a. The risk of perforation of the esophagus associated with balloon dilatation may be as high as 10%
b. An anti-reflux procedure should be universally performed for any operative myotomy
c. Successful relief of dysphagia can be achieved in up to 90% of patients with a single pneumatic
dilatation
d. Thoracoscopic myotomy is associated with significantly poorer results than the open procedure
e. Prospective randomized studies and retrospective data appear to support a surgical approach for
achalasia
Answer: a, e
The mainstay of treatment in achalasia is either balloon dilatation or surgery. Balloon dilatation has an
advantage that it can be performed as an outpatient and has minimal recovery time. It is less likely to
be effective than surgical treatment, and frequently needs to be repeated. The risk of perforation of the
lower esophagus is higher with this procedure than with any other form of esophageal instrumentation
and varies from 2–10%. The risk of gastroesophageal reflux following dilatation is not known, but
symptomatically the risk appears to be low.
All surgical procedures employ a variant of Heller’s myotomy, in which the circular muscle of the lower
esophagus is divided. In the United States, most myotomies are carried out through the chest, but the
abdominal approach is favored in Europe. Regardless of the route chosen, four key principles are
important, namely: 1) adequate myotomy, 2) minimal hiatal disturbance, 3) anti-reflux protection
without creation of obstruction, and 4) prevention of rehealing. The advent of minimally invasive surgery
has led to the development of thorascopic and laparoscopic myotomy, and these are now being
extensively performed with comparable results to open surgery. There is broad agreement that if a
myotomy is performed through the abdomen, an anti-reflux procedure should be added, and that a full
Nissen wrap, however floppy, leads to long-term failure. When approached through the chest, there is
controversy about the need for an anti-reflux procedure, as it is claimed that less hiatal disturbance and
more limited myotomy is possible by this route. Thoracoscopic myotomy, with enhanced view, enables a
more precise determination of the distal myotomy and therefore may not require a anti-reflux
procedure.
A single pneumatic dilatation achieves adequate relief of dysphasia and pharyngeal regurgitation in
about 60% of patients. Repetitive dilatations increase this figure to about 70%. Only one controlled
randomized study comparing the two modes of therapy has ever been performed. The results of this
study as well as a number of retrospective studies would appear to support operative myotomy as the
initial treatment of choice.

67. Which of the following statement(s) is/are true concerning the surgical anatomy of the esophagus?

a. Surgical exposure of the cervical esophagus is best gained via the right neck
b. Spontaneous esophageal perforation tends to be associated with leakage into the left chest
c. Access to the entire thoracic esophagus can be obtained only via the left chest
d. The lower esophageal sphincter can be recognized distinctly by inspection of the gastroesophageal
junction
Answer: b

A detailed knowledge of the relations of the esophagus is essential for the surgeon to be able to identify
the site and significance of lesions seen by indirect studies such as endoscopy, contrast radiography, and
CT scanning, as well as the safe performance of surgical procedures. The cervical esophagus is about 5
cm long. It begins at the level of C6 and extends to the lower border of T1, curving slightly to the left in
its descent. Consequently, although the surgical approach to this portion of the esophagus may be from
either side of the neck through an incision along the anterior border of the sternocleidomastoid muscle,
the left side is chosen if possible. Above the level of the tracheal bifurcation, the esophagus moves to
the right of the descending aorta. It then moves to the left, passes behind the tracheal bifurcation and
the left main bronchus and descends to the diaphragm. In the lower third, the esophagus courses
anteriorly and to the left to pass through the diaphragmatic hiatus. The lower esophagus is covered only
by a flimsy mediastinal pleura on the left, and it is this portion which is most commonly the site of
spontaneous perforation in Boerhaave’s syndrome. In general, the lower esophagus is most easily
approached through the left chest, but access to the supra-aortic esophagus is restricted. Thus, a left
thoracotomy is most useful for performing procedures involving the lower esophagus. However, access to
the entire thoracic esophagus can be obtained only from the right chest. This incision, however, limits
access to intraabdominal organs by the position of the liver and therefore normally requires a separate
upper abdominal incision. The abdominal esophagus begins as the esophagus enters the abdomen
through the diaphragmatic hiatus. It is surrounded by a fibroelastic membrane, the phrenoesophageal
ligament which arises from the subdiaphragmatic fascia. The lower limit of the pharyngoesophageal
membrane anteriorly is marked by a prominent fat pad, which corresponds to the gastroesophageal
junction. The lower esophageal sphincter (LES) is a zone of high pressure 3–5 cm long at the lower end of
the esophagus. Although it does not correspond to any macroscopic anatomical structure, its function
appears to be related to the microscopic architecture of the muscle fibers.

68. Which of the following statement(s) is/are correct concerning the patient whose barium esophogram
is shown below?
a. The patient’s complaint would be primarily chest pain and to a lesser degree dysphagia
b. The pathognomic feature of manometry is the presence of prolonged high amplitude waves
c. The patient will likely experience nutritional problems
d. The first line of treatment for this patient is surgical myotomy
Answer: a, b

The barium esophogram of these two patients shows diffuse esophageal spasm resulting in a cork screw
esophagus with multiple contractions. See Fig. 18-35. These primary motor disorders are characterized
by substernal chest pain. In the nutcracker variety, as demonstrated in this x-ray, the pain is central
crushing pain with no relation to food ingestion and differs from angina in that it more frequently comes
on at rest. Dysphagia or classic heartburn may be present but tend to be overshadowed by the chest
pain. Barium radiography and endoscopy are generally not helpful. The pathognomic feature of
manometry is the presence of prolonged high amplitude waves, with a peak greater than 180 mm Hg.
Diffuse esophageal spasm and nutcracker esophagus are benign conditions which rarely cause nutritional
problems and do not lead to life-threatening complications. For this reason, symptom control is the only
significant goal of treatment. Medical treatment for diffuse esophageal spasm and nutcracker esophagus
is focused on abolishing strong simultaneous contractions and generally employs calcium channel
blocking agents or long-acting nitrates. Surgery for these conditions are generally only considered when
medical treatment is ineffective.

69. Which of the following statement(s) is/are true concerning tracheoesophageal fistulas?

a. The majority of acquired tracheoesophageal fistulas are due to malignant disease


b. A water-soluble contrast esophogram should be obtained for diagnosis
c. Malignant tracheoesophageal fistulas represent one of the few indications for an endoesophageal
prosthesis
d. A benign tracheoesophageal fistula from an endotracheal intubation injury often requires a
thoracotomy for repair
Answer: a, c

Ninety percent of acquired fistulas between the esophagus and tracheobronchial tree in adults are the
result of malignant disease. Tracheoesophageal fistulas complicate the course of disease in about 5% of
patients who have esophageal carcinoma. Nearly 80% of patients with malignant tracheoesophageal
fistulas die within three months of the onset of symptoms and in 85% of these patients, the cause of
death is aspiration pneumonia, not distant metastatic disease. For the most part, malignant
tracheoesophageal fistula represents incurable disease for which resection carries significant mortality
and is seldom indicated. Palliative relief of recurrent aspiration is the aim of therapy. Effective
occlusion of the fistula may be achieved by insertion of one of a variety of available endoesophageal
endoprostheses. These tubes are placed into the esophagus with the aid of an esophagoscope and may
occlude the esophageal side of the fistula sufficiently to allow swallowing of liquids without aspiration
into the tracheobronchial tree. More recently, expandable metal stents have been used successfully in
the treatment of malignant tracheoesophageal fistulas.
Nonmalignant fistulas result from the erosion by contiguous infected subcarinal mediastinal lymph
nodes; trauma; late sequelae of chronic mid-esophageal traction diverticulum; or erosion by an
endotracheal or tracheostomy tube cuff in a patient requiring prolonged ventilatory support. Small
fistulas, such as resulting from an endotracheal intubation injury, are approached through a cervical
collar or oblique incision anterior to the sternocleidomastoid muscle. Although such cuff injuries usually
produce circumferential tracheal damage which necessitates a tracheal resection, this can also be
performed through a cervical collar incision.

70. Esophageal cysts arise as outpouchings of the embryonic foregut. Which of the following statement(s)
is/are true concerning esophageal cysts?

a. The cyst lining will be lined only by stratified squamous epithelium


b. Most esophageal cysts cause symptoms in the first year of life
c. An asymptomatic esophageal cyst can be managed conservatively
d. The diagnosis of an esophageal cyst is usually made radiographically
Answer: b, d

Embryologically, the esophagus is lined by simple columnar ciliated epithelium, which is eventually
replaced by stratified squamous epithelium. The esophageal cyst may therefore contain both of these
types of epithelium as well as fat and smooth muscle. The esophageal duplication cyst is a variation of
the foregut cyst, extends along the length of the thoracic esophagus, and is lined by squamous
epithelium. More than 60% of esophageal cysts cause either respiratory or esophageal symptoms in the
first year of life. Those located in the upper third of the esophagus tend to present in infancy, while the
lower-third cyst may be asymptomatic initially and present later in childhood. Adults present with
dysphasia, choking, retrosternal pain when previously asymptomatic cysts enlarge as the result of
bleeding or infection. The diagnosis of an esophageal cyst can usually be made on the basis of atypical
radiographic appearance. The PA and lateral chest x-ray, barium esophogram, and in some cases a CT
scan, will confirm the diagnosis in almost all patients. Because esophageal cysts have a predilection for
bleeding, ulceration, perforation, and infection, excision is generally recommended. This can generally
be achieved with low morbidity by an extramucosal resection.

71. Which of the following statement(s) is/are true concerning infectious esophagitis?

a. Candida albicans is not normally found in the mouth but results from the overgrowth of this fungus in
patients on broad spectrum antibiotics
b. Candida esophagitis is usually self-limited and is seldom associated with chronic problems
c. Systemic therapy is seldom indicated
d. Small ulcers on barium esophogram in a transplant patient complaining of dysphagia and odynophagia
are likely due to herpes simplex viral infection
Answer: d

Chronic debilitation, immunosuppression, and prolonged use of antibiotics predisposes the development
of infectious esophagitis with candida albicans being the most common cause. Candida albicans is a
fungus that normally is a commensal inhabitant of the mouth, oral pharynx, and GI tract. This fungus
may become pathogenic in patients who are severely debilitated or immunosuppressed. In recent years,
the use of broad spectrum antibiotics, immunosuppression in organ transplant patients, and the wide use
of chemotherapeutic agents have resulted in an increased number of patients with monilial esophagitis.
As the disease progresses, transmural invasion of the esophageal wall occurs. Although the esophagitis
can be controlled with antifungal therapy, if the patient survives the underlying illness, chronic stricture
formation may result after healing. Minimally compromised patients with mild monilial esophagitis
should receive oral nystatin suspension as a primary treatment. More immunosuppressed patients or
those with severe cases warrant high doses of fluconazole and ketoconazole. Intravenous fluconazole or
amphotericin B are utilized in granulocytopenic patients.
Viral esophagitis is the second most common cause of infectious esophagitis. Herpes simplex viral
infection is the most common infection in the immunosuppressed transplant patient. Characteristically,
viral esophagitis produces mucosal ulceration with patients presenting with dysphasia and odynophagia.
The esophageal ulcers are characteristically small (< 1.5 cm). The diagnosis is established endoscopically
by biopsy, brushings, and washings for cytology, histology, and viral culture. The infection usually
responds well to treatment with acyclovir.

72. Which of the following statement(s) is/are true concerning the pathology of squamous cell carcinoma
of the esophagus?

a. Carcinoma in situ will gradually progress to invasive squamous cell carcinoma over a period of two to
four years
b. The most common location for squamous cell carcinoma of the esophagus is the upper and mid-
thoracic segment
c. Esophageal carcinoma tends to be multifocal
d. Macroscopically, ulcerative lesions with extensive infiltration of the adjacent esophageal wall are
most common
e. Lymph node metastases are present in at least 75% of patients at the time of initial diagnosis
Answer: a, b, d, e

Pathologically, esophageal carcinoma occurs over a spectrum that ranges from the early lesion
(carcinoma in situ), which is limited to the mucosa, to the more advanced form, in which the tumor
penetrates the muscle layers of the esophagus and beyond. Carcinoma in situ typically is found in
patients between 40 and 50 years of age and gradually progresses to invasive squamous cell carcinoma
over two to four years. Using the arbitrary division of the esophagus, 8% of squamous cell carcinomas
occur in the cervical esophagus, 55% in the upper and mid-thoracic segments, and 37% in the lower
thoracic segment which extends to the GE junction. Macroscopically, 60% of squamous cell carcinomas of
the esophagus are fungating intraluminal growths, 25% of ulcerative lesions are associated with extensive
infiltration of the adjacent esophageal wall, and 15% are infiltrating. Esophageal carcinoma tends to be
multi-focal, and a patient who survives treatment of one carcinoma has at least twice the risk of
developing a second primary esophageal neoplasm than the normal population.
Esophageal carcinoma is notorious for its aggressive biologic behavior. Mediastinal, supraclavicular, or
celiac lymph node metastases are present in at least 75% of patients with esophageal cancer at the time
of initial diagnosis. Unfortunately, when lymph node metastases are present, five-year survival is only
3%, compared with 42% when there is no lymph node spread.

73. A 54-year-old woman experiences pain in both the anterior and posterior left chest and the
epigastrium following balloon dilatation performed for achalasia. Which of the following statement(s)
is/are true concerning this patient’s diagnosis and management?

a. A normal chest x-ray will rule out an esophageal perforation


b. Barium should never be used in performance of a contrast study with a diagnosis of esophageal
perforation
c. Conservative, nonoperative treatment may be indicated
d. If surgical repair is necessary, the patient should undergo esophagomyotomy and a partial gastric
fundoplication
Answer: c, d

It is axiomatic that pain or fever after esophageal instrumentation or operation is indicative of an


esophageal perforation until proven otherwise. It is an indication for immediate contrast esophogram.
Because the morbidity and mortality rates associated with esophageal perforation are directly related to
the time interval between diagnosis of the injury and its repair or drainage, an aggressive attitude
toward diagnosing the perforation must be adopted. When the diagnosis is considered, a water-soluble
contrast agent should be administered. If this study is negative, dilute barium should be administered.
Barium is relatively inert, and the fear of barium extravasating in the mediastinum through the site of
injury and producing a severe reactive mediastinitis is unfounded. A chest x-ray may help confirm the
diagnosis by demonstrating air in the soft tissues of the neck or mediastinum or a hyrdo-or
pneumothorax. A normal chest x-ray, however, does not rule out an esophageal perforation.
Although most esophageal perforations require operative intervention, selected patients may be
managed nonoperatively with cessation of oral intake, administration of antibiotics, and intravenous
hydration until the disruption heals or the small contained cavity begins to decrease in size. The usual
clinical settings for such perforations that are encountered are cervical esophageal tears caused by
esophagoscopy; intramural dissection that has occurred during dilatation of a stricture or pneumatic
dilatation for achalasia, and an asymptomatic esophageal anastomotic disruption discovered on a routine
postoperative contrast study. Perforations complicating pneumatic dilatation for achalasia occur in 4% to
6% of patients, and most are small and well managed medically with antibiotics and intravenous
hyperalimentation. If operation is required for suture repair of the perforation, an esophagomyotomy to
relieve the distal obstruction, and a partial fundoplication to buttress the tear should be performed if
possible.

74. Which of the following statement(s) is/are correct concerning the diagnostic studies for esophageal
carcinoma?

a. A chest and upper abdominal CT scan is useful for both staging and predicting resectability
b. A barium swallow is an unnecessary test in a patient with dysphagia
c. Bronchoscopy should be performed in all patients with carcinoma of the upper and middle thirds of
the esophagus
d. Bone and brain scans should be obtained routinely to rule out distant metastasis
e. Endoscopic ultrasound is a potentially sensitive examination for the staging of esophageal cancer
Answer: c, e

A barium swallow examination is the first study that should be obtained in a patient who complains of
dysphagia. Tumors of the cervical esophagus are difficult to identify by barium swallow examination and
carcinoma of the cardia may be confused with achalasia or esophageal spasm. Nevertheless, the barium
swallow examination localizes obvious esophageal pathology in preparation for subsequent
esophagoscopy and allows the endoscopist to predict the level at which the tumor is located and the
area which requires the most careful examination. The chest and upper abdominal CT scan is now the
standard radiographic technique for staging esophageal carcinoma. Esophageal wall thickness, regional
adenopathy or pulmonary, liver, adrenal or distant nodal metastasis can be identified. Although CT is
suggested to have a role in evaluating resectability of esophageal carcinoma, it is particularly limited in
its ability to detect invasion of the gastric cardia or aortic invasion. Bone scan is not warranted unless
the patient has specific complaints suggesting that bone metastases exists. Similarly, routine brain scans
are not indicated as brain metastases from carcinoma of the esophagus are uncommon. Bronchoscopy
should be performed in patients with carcinoma of the upper and middle thirds of the esophagus to
exclude invasion of the posterior membranous trachea or mainstem bronchi, which precludes a safe
esophagectomy. Endoscopic ultrasound is being used with increasing frequency as an adjunct to the
standard radiologic and endoscopic assessment of esophageal disease. It offers the potential for more
sensitive staging of esophageal carcinoma by detecting the depth of invasion and the presence of
abnormal mediastinal adenopathy.

75. Which of the following conditions are associated with the development of esophageal carcinoma?

a. Caustic esophageal stricture


b. Achalasia of the esophagus
c. Plummer-Vinson syndrome
d. Esophageal diverticula
Answer: a, b, c, d

Chronic irritation of the esophageal mucosa by a variety of noxious stimuli (alcohol, tobacco, hot foods
and liquids) eventually may lead to the development of esophageal carcinoma. A variety of other
esophageal lesions have a recognized premalignant nature. The patient who survives the initial injury
long enough to develop a caustic esophageal stricture has a 1000-fold increased risk of developing
carcinoma compared with the normal population. Ten to 12% of patients with achalasia of the esophagus
who are observed 15 years or more develop esophageal carcinoma. This is thought to be related to the
irritating effects of the fermenting intraesophageal contents on the adjacent esophageal mucosa.
Plummer-Vinson syndrome is a premalignant esophageal condition. Patients with this syndrome are
typically elderly women who have cervical dysphasia and iron deficiency anemia. About 10% of patients
will develop squamous cell carcinoma of the hypopharynx, oral cavity or esophagus. Finally, there have
been isolated reports of esophageal carcinomas found incidentally within esophageal diverticula,
presumably as the result of the irritating effects on the mucosa of stagnant, putrefying food within the
pouch. Esophageal diverticula are therefore regarded as premalignant esophageal lesions although this
occurrence is extremely rare.

76. Which of the following statement(s) is/are correct concerning the options for resection of
esophageal carcinoma?

a. The development of reflux esophagitis seldom occurs following intrathoracic resection due to the
limited life expectancy of these patients
b. Transhiatal esophagectomy, although conceptually sound, is not technically possible in most patients
with esophageal carcinoma
c. Transhiatal resection, although less morbid, has unfavorable survival statistics compared to
transthoracic resection
d. Radical transthoracic esophagectomy with en bloc dissection of continuous lymph node bearing tissues
has not been shown to improve survival over transhiatal esophagectomy
Answer: d

For most patients with localized esophageal carcinoma, resection provides the most effective and
reliable palliation of dysphagia. The rational surgical approach to distal esophageal carcinoma has been
a left thoracoabdominal incision. Tumors involving the mid-esophagus have been resected either through
a thoracoabdominal or separate thoracic and abdominal incision, and a high thoracic esophagogastric
anastomosis is performed. The major disadvantages of this technique are the necessity of a thoracotomy
in debilitated patients with esophageal obstruction as well as the disastrous complications following
intrathoracic esophageal anastomotic leak. Although recent results have shown improved operative
mortality rates, the operation can still be associated with significant morbidity and mortality. A further
disadvantage of the standard intrathoracic esophagogastric anastomosis is inadequate long-term relief of
dysphagia either due to tumor recurrence at the anastomotic suture line or due to the development of
reflux esophagitis above the anastomosis. Although it has been long taught that the patient with
esophageal carcinoma does not live long enough to develop reflux esophagitis after a low intrathoracic
esophagogastric anastomosis, this is clearly not the case, and the development of reflux in these
patients can produce not only severe pyrosis and reflux symptoms, but also dysphagia from benign
stenosis.
During the last two decades, the technique of transhiatal esophagectomy without thoracotomy has been
popularized as an operation that minimizes the factors responsible for poor results from traditional
transthoracic esophageal resection and reconstruction. In experienced hands, transhiatal esophagectomy
without thoracotomy is possible in over 90% of patients. The survival data is comparable to those
obtained in most series of transthoracic resection with results usually demonstrating decreased
postoperative morbidity and mortality. Although conceptually radical transthoracic esophagectomy with
en bloc dissection of contiguous lymph node bearing tissues would appear to offer a better “cancer
operation” than transhiatal esophagectomy with no formal lymph node dissection, current survival
results are not statistically different. These data suggest that survival after resection for esophageal
carcinoma is more a function of the extent and stage of the tumor rather than the size of the specimen
or the number of lymph nodes removed.

77. The incidence of adenocarcinoma of the esophagus is increasing at a very rapid rate, which is largely
the result of the growing prevalence of adenocarcinoma arising in Barrett’s mucosa. Which of the
following statement(s) is/are true concerning adenocarcinoma of the esophagus?

a. Barrett’s mucosa with specialized columnar epithelium characterized by veliform folds, lined by
secreting columnar and goblet cells has the highest association with carcinoma of the esophagus
b. Less than 5% of patients with Barrett’s mucosa will harbor adenocarcinoma
c. Severe dysplasia of Barrett’s mucosa requires frequent reexamination and biopsy
d. Adenocarcinoma of the esophagus has a less aggressive behavior than squamous cell carcinoma
Answer: a

It is estimated that patients with Barrett’s esophagus are 40 times more likely to develop
adenocarcinoma than the general population. The true incidence of Barrett’s esophagus in the general
population is unknown, but it is estimated that adenocarcinoma arises in up to 8% to 15% of patients with
columnar epithelium lined esophagus. Of the three characteristic histologic patterns for Barrett’s
mucosa, the specialized or intestinal type of metaplasia which is characterized by veliform folds lined by
a single layer of glycoprotein secreting columnar cell and mucous-secreting goblet cells has the highest
association with carcinoma. Dysplasia occurs to varying degrees in Barrett’s mucosa and is clearly a
premalignant lesion. Severe dysplasia is almost always associated with carcinoma in situ and mandates
aggressive therapy. As is true of squamous cell carcinomas, esophageal adenocarcinoma has an
aggressive biologic behavior that is characterized by frequent transmural invasion and lymphatic spread.

78. Which of the following statement(s) is/are true concerning esophageal diverticula?

a. A Zenker’s diverticulum characteristically occurs in older patients


b. Mediastinal granulomatous disease usually results in a mid-esophageal traction diverticulum which is
usually asymptomatic
c. An epiphrenic diverticulum that presents to the right of the esophagus should be managed via left
thoracotomy
d. Minimally symptomatic epiphrenic diverticula should not be operated upon
Answer: b, c, d

An esophageal diverticulum is an epithelial-lined mucosal pouch that protrudes from the esophageal
lumen. Most esophageal diverticula are acquired, and occur predominantly in adults. The
pharyngoesophageal (Zenker’s diverticulum) is the most common esophageal diverticulum and typically
occurs in patients between 30 and 50 years of age. Mediastinal granulomatous disease (e.g., tuberculosis
or histoplasmosis) is the most common cause of mid-esophageal traction diverticulum. This type of
diverticulum is much smaller than the pulsion diverticulum and has a characteristic blunt tapered tip
that points toward the adjacent subcarinal or peribronchial lymph nodes to which it adheres. It is
typically diagnosed as an incidental finding on a barium esophogram and is almost always asymptomatic.
No specific treatment is indicated.
An epiphrenic or supradiaphragmatic diverticulum occurs within the distal 10 cm of the thoracic
esophagus as a pulsion diverticulum that arises because of abnormally elevated intraluminal esophageal
pressure. Although many patients are asymptomatic at the time of diagnosis on barium esophogram,
others have symptoms from the frequently associated esophageal conditions: hiatal hernia, diffuse
esophageal spasm, achalasia, reflux esophagitis and carcinoma. Pouches smaller than 3 cm and causing
little or no symptoms require no treatment. Severe dysphagia, chest pain, or an anatomically dependent
or enlarging pouch are indications for repair. The surgical approach to an epiphrenic diverticula is
through a left 6th or 7th interspace posterolateral thoracotomy. This is the case even for diverticula that
present to the right of the esophagus.

79. Which of the following statement(s) is/are true concerning caustic injury to the esophagus?

a. Alkaline injury is more destructive than acid injury


b. Acid ingestion is not injurious to the stomach due to its normal acidic pH
c. Ingested caustic agents rapidly pass through the esophagus and stomach into the small intestine
d. Unless perforation occurs, clinical manifestations resolve quickly with initial clinical improvement
noted
e. Children are less likely to form a late esophageal stricture than adults
Answer: a, d, e

Caustic injury occurs in two broad categories of patients, children younger than 5 years of age who
accidentally swallow these agents, and adults who are attempting suicide. The most common agents
responsible for caustic esophageal injuries are alkalis, acids, bleach and detergents. Ingestion of
detergents and bleaches virtually always cause only mild esophageal irritation which heals without
significant adverse sequelae. Acids and alkalis may have devastating effects which range from acute
multi-organ necrosis and perforation to chronic esophageal and gastric strictures. Alkalis are more
destructive, producing liquefaction and necrosis which almost insures deep penetration whereas acids
usually cause coagulation necrosis that, in part, limits the depth of injury. Liquid alkali preparations
have prolonged contact with the mucosa of the esophagus and stomach due to its high viscosity. In
addition, ingested acids typically pass through the esophagus quickly, producing major gastric injury with
relative sparing of the esophagus. In response to either ingested acid or alkali, reflex pyloric spasm
occurs, with resultant pooling of these agents in the gastric antrum.
The clinical manifestations of caustic ingestion are directly related to the amount and character of the
agent ingested. When esophageal or gastric perforation results from caustic ingestion, patients
demonstrate progressive severe sepsis and hypovolemic shock until appropriate resuscitative measures
are instituted. In the absence of gastric or esophageal perforation, the acute clinical manifestations
typically resolve within days, with clinical improvement lasting for weeks. After this, symptoms due to
either esophageal or gastric stricture begin to form. Most adults who ingest liquid alkali will develop
severe esophageal and usually gastric injury that results in stricture formation. Children, with usually
more limited exposure from accidental ingestions, are less likely to have severe injuries.

80. Which of the following statement(s) is/are true concerning nonresectional therapy for esophageal
carcinoma?

a. Radiation therapy can be associated with five-year survival rates equal to surgery
b. Esophageal intubation to provide palliation for esophageal cancer is associated with minimal
morbidity and mortality
c. Endoscopic laser fulguration is successful in up to 75% of patients
d. There is little or no role for surgical bypass for unresectable esophageal carcinoma
Answer: c, d

Therapy of esophageal carcinoma is influenced by the knowledge that in most of these patients, local
tumor invasion or distant metastatic disease preclude cure. While squamous cell carcinoma is generally
regarded as a radiosensitive and therefore potentially curable tumor, radiation therapy has not achieved
cure in most patients. Although “curative” super voltage radiation techniques have been employed, the
average five-year survival after such treatment is between 6 and 10% in most series. This is somewhat
poorer than five-year survival rates after resection which usually range between 10 and 15%. A variety of
endoesophageal tubes have been used for palliation in patients with esophageal carcinoma. Basically,
these tubes are divided into two types: the pulsion tubes, which are pushed through the tumor with the
aid of an esophagoscope; and the traction or pull-through tubes, which are pulled into place by
downward traction through a gastrostomy. As in the case with many conceptually simple procedures,
implementation in the clinical setting is problematic. Transoral esophageal intubation is associated with
an overall mortality of 14% and a complication rate of 25%, the latter due to perforation of the
esophagus, migration of the tubes, or obstruction of the tubes by food or tumor overgrowth. More
recently, a variety of expandable intraesophageal metallic stents have been used to achieve palliation in
patients with unresectable esophageal carcinoma. Additional expertise with this technique is being
acquired through a multi-institutional trial currently underway in the United States. Endoscopic laser
fulguration of esophageal carcinoma has been used to achieve temporary relief of the esophageal
obstruction in patients with unresectable tumors. Generally, multiple sessions are required to resect
sufficient tumor to achieve adequate lumen, but functional success with restoration of a comfortable
volume can be achieved in 75–80%. Although a variety of surgical procedures such as substernal gastric or
colon bypasses have been developed as palliative internal bypasses of unresectable esophageal
carcinoma, the limited survival in these unresectable patients can be high and the mortality rates,
between 15 and 25%, do not currently justify their use.

81. Benign tumors of the esophagus are rare constituting less than 1% of esophageal neoplasms. Which of
the following statement(s) is/are true concerning benign esophageal neoplasms?

a. Most esophageal polyps are located just above the gastroesophageal junction
b. Malignant degeneration of leiomyomas of the esophagus is a frequent occurrence
c. An asymptomatic leiomyoma can be safely observed and followed with periodic barium esophograms
and endoscopic ultrasonography
d. Most leiomyomas of the esophagus require esophagectomy
Answer: c

Leiomyomas represent the most common benign intramural esophageal tumor and characteristically
occur in patients between 20 and 50 years of age. More than 80% of esophageal leiomyomas occur in the
middle and lower thirds of the esophagus. Tumors less than 5 cm in size rarely cause symptoms. When
larger than this, dysphagia, retrosternal pressure and pain are the common complaints. Bleeding more
often occurs with the malignant form of the tumor, leiomyosarcoma. Malignant degeneration of
leiomyomas is exceedingly rare. An asymptomatic leiomyoma or one discovered incidentally on a barium
swallow examination can be safely observed and followed with periodic barium esophograms and
endoscopic ultrasonography. Although excision of the esophageal mass provides the only definitive tissue
diagnosis, the characteristic radiographic appearance, slow growth rate, and low risk of malignant
degeneration and the ability to follow leiomyomas with endoscopic ultrasonography justify conservative
management. Tumors that are symptomatic or larger than 5 cm in diameter should be excised. Tumors
of the middle third of the esophagus are approached through a right thoracotomy, while those in the
distal third are approached through a left thoracotomy. Once the esophagus is encircled and the tumor
located, the overlying longitudinal muscle is split in the direction of its fibers. The tumor is then gently
dissected away from the contiguous underlying submucosa and adjacent muscle. When enucleation of
the tumor is complete, the longitudinal esophageal muscle is reapproximated, although a large
extramucosal defect may be left without complication. Giant leiomyomas of the cardia and adjacent
stomach may require esophageal resection for their removal. When resection is complete, leiomyomas
virtually never recur.
Benign polyps of the esophagus are rare and typically arise in the cervical esophagus. Most are seen in
older men and are frequently attached to the cricoid cartilage. Histologically, they are composed of
fibrovascular tissue with varying amounts of associated fat.

82. In an effort to improve survival following esophageal resection, trials of multi-modality therapy in
combination with surgery have been completed. Which of the following statement(s) is/are true
concerning such treatment?

a. Therapy appears to be indicated in squamous cell carcinoma but not adenocarcinoma


b. No residual carcinoma may be found in the resected specimen in up to 20% of patients
c. Nonrandomized trials would suggest improved survival compared to patients receiving surgery alone
d. Perioperative morbidity is increased due to preoperative radiation and chemotherapy
Answer: b, c,

Combined preoperative chemotherapy and radiation therapy before transhiatal esophagectomy for
carcinoma has provided encouraging survival statistics. The treatment consists of preoperative
chemotherapy with three weeks of cisplatin, vinblastine, and 5-fluorouracil, concurrent with 3750 to
4500 cGy of radiation therapy. Although hematologic toxicity and radiation esophagitis are common and
preoperative deaths due to bone marrow suppression can occur, there is no increase in perioperative
morbidity when compared to patients with no preoperative therapy. In one study, 24% of patients had no
residual carcinoma in the resected specimen (T0, N0 status). At mean follow-up of 36 months, the mean
survival in this series was 29 months, a clear improvement over the 12-month median survival time with
transhiatal esophagectomy alone in historical controls.
83. Which of the following statement(s) is/are correct concerning the management of a patient with a
caustic esophageal or gastric injury?

a. Corticosteroids should be administered immediately


b. Complete endoscopic examination of the esophagus and stomach should be completed
c. Patients requiring operative intervention are best explored through the abdomen
d. If organ resection is indicated, restoration of alimentary continuity should be deferred until the
patient has recovered from the acute insult
e. In patients with esophageal stricture following second and third degree burns, dilatation therapy
should be instituted as soon as possible after the injury
Answer: b, c, d

Acute caustic ingestion is indication for hospitalization. Initial management centers on stabilizing the
patient and assessing the severity of injury. Oral intake should be withheld and hypovolemia corrected
with intravenous fluids. Careful observation for evidence of airway obstruction is mandatory. Broad
spectrum antibiotics are indicated once the diagnosis of substantial esophageal injury has been
established to diminish the risk of pulmonary infection from aspiration as well as bacterial invasion
through the damaged esophageal wall. Although corticosteroids have been advocated in the acute phase
of caustic ingestion to minimize subsequent stricture formation, their efficacy has not been established.
Furthermore, because steroids may mask signs of sepsis, visceral perforation, and impair healing, their
use in caustic esophageal injury is potentially deleterious and therefore is not recommended.
A contrast esophogram is the best way to make the diagnosis of esophageal perforation and should be
performed if the diagnosis is suspected at the time of admission or in subsequent followup.
Esophagoscopy should be performed soon after admission to establish whether significant esophageal
injury has occurred and to permit grading of the severity of injury. Although in the past it was taught
that the endoscope should not be advanced beyond the first burned area, more recently complete
examination of the esophagus and stomach has been recommended, especially if severe burns are not
detected proximally. The use of a pediatric endoscope and adequate sedation can allow this procedure
to be accomplished safely. Patients with caustic liquid ingestion that necessitate operative intervention
are generally best explored through the abdomen. This approach permits assessment of the injury to the
intraabdominal organs as well as resection of areas of full thickness gastric necrosis. Although only the
lower esophagus is well visualized through the diaphragmatic hiatus, if an esophageal resection is
required, transhiatal esophagectomy without thoracotomy is readily performed by the addition of a
cervical incision. When esophageal gastric resection for acute caustic injury is required, restoration of
alimentary continuity should be deferred until the patient has recovered from the acute insult and the
development of chronic stricture formation and retained organs can be evaluated. Esophageal stricture
formation following second and third degree burns is the rule, and dilatation therapy has been the
traditional therapy for chronic caustic esophageal strictures. It is important that dilatation not be
instituted until at least six to eight weeks after the injury, when reepithelialization is complete, in order
to minimize the risk of perforation.

84. In regard to the arterial blood supply to the stomach, which of the following statement(s) is/are
true?

a. The right gastric artery, a branch of the superior mesenteric artery, supplies the gastric antrum
b. Because of rich intramural collaterals, gastric viability may be preserved after ligation of all but one
major artery
c. In cases of celiac artery occlusion, gastric viability is maintained collaterally via pancreaticoduodenal
arcades
d. The left gastroepiploic artery is a branch of the celiac trunk
Answer: b, c

The stomach is an extremely well-vascularized organ, supplied by 5 major arterial distributions and
protected from ischemia by rich intramural and extramural collaterals. The left gastric artery and right
gastric artery, derived from the celiac distribution, supply the lesser curvature of the stomach. The right
gastroepiploic artery, derived from the gastroduodenal artery, and the left gastroepiploic artery, from
the splenic artery, traverse the greater curvature. The area adjacent to the spleen receives multiple
short gastric arterial branches. In instances of celiac arterial occlusion, the superior mesenteric artery
supplies the stomach collaterally via the pancreaticoduodenal arcades which connect with the
gastroduodenal artery. The stomach may be widely mobilized for use in reconstructive procedures, for
example, during trans-hiatal esophagectomy. Advantage is taken of the abundant blood supply and
collaterals of the stomach during mobilization; gastric viability is usually preserved if one major arterial
supply is preserved.

85. At a cellular level, the major stimulant(s) of acid secretion by the gastric parietal cell is/are:

a. Histamine
b. Prostaglandin E2
c. Acetylcholine
d. Gastrin
e. Norepinephrine
Answer: a, c, d

The three major stimulants of acid secretion by the parietal cell are acetylcholine, gastrin, and
histamine. Acetylcholine is released from cholinergic nerve endings in close proximity to parietal cells
and binds to muscarinic receptors. Cholinergic stimulation of parietal cells is coupled to hydrolysis of
membrane-associated lipids (termed phosphatidylinositides) and leads to increases in intracellular
calcium. Histamine is released from mast cells in the lamina propria and reaches parietal cells by
diffusion. Histamine occupies H2 receptors that may be selectively blocked by agents such as cimetidine.
Histamine stimulation of parietal cell acid secretion is mediated by a cyclic AMP-dependent pathway.
Gastrin is delivered to the fundic mucosa by the systemic circulation from its source in the antrum and
duodenum. Like acetylcholine, gastrin causes increases in membrane phosphoinositol turnover and
increases intracellular calcium.
Activation of parietal cells by acetylcholine, gastrin or histamine can be blocked by somatostatin. Local
release of somatostatin is physiologically important in modulating postprandial gastric acid secretion.
Prostaglandin E2 and its synthetic derivatives are potent inhibitors of histamine-stimulated acid
secretion.

86. Which of the following statement(s) regarding the vagus nerves is/are true?

a. The right and left vagus nerves derive from a nerve plexus inferior to the tracheal bifurcation
b. The posterior vagus nerve is closely applied to the intrathoracic esophagus
c. The anterior vagus supplies a hepatic division which passes to the right in the lesser omentum
d. Approximately 90% of vagal fibers are afferent, transmitting information from the gastrointestinal
tract to the central nervous system
e. The vagus nerves transmit gastroduodenal pain sensations associated with peptic ulceration
Answer: a, c, d

The left and right vagus nerves are formed from a periesophageal nerve plexus between the tracheal
bifurcation and the diaphragm. As they pass through the esophageal hiatus, the anterior vagus is closely
applied to the esophagus; the posterior vagus lies intermediate in position between the esophagus and
the aorta. The anterior vagus supplies the hepatic division which provides parasympathetic innervation
to the liver and biliary tract. The hepatic division is usually easily seen in the thin gastrohepatic
omentum and is constant in location. The hepatic division is a useful anatomic landmark in vagotomy
procedures. The posterior vagus nerve supplies fibers to the celiac division. After giving off hepatic and
celiac divisions, both anterior and posterior vagus nerves supply branches to the gastric wall.
Surprisingly, only 10% of vagal fibers are efferent, secretomotor fibers; almost 90% are afferent.
Sensations of gastric pain are carried in sympathetic fibers, and vagotomy does not alter perception of
painful gastric conditions or stimuli.

87. Important stimulants of gastrin release from endocrine cells in the antrum include:

a. Acidification of the antral lumen


b. Small peptide fragments and amino acids from luminal proteolysis
c. Locally released somatostatin
d. Dietary fats
Answer: b

Gastrin is processed to 34-and 17-amino acid forms in endocrine cells in the gastric antrum. In addition
to well-recognized stimulatory actions on gastric acid secretion, gastrin promotes mucosal growth of the
gastric fundus and small intestine. The most important stimulant of gastrin release is a meal. Small
peptide fragments and amino acids that result from intragastric proteolysis are the food components
that stimulate gastrin release. Ingested fats and carbohydrates have no significant effect. In this regard,
intraluminal pH strongly affects gastrin secretion. If intragastric pH is maintained above 3 after ingestion
of a meal, gastrin release is strongly potentiated. Pernicious anemia and atrophic gastritis, which
produce chronic achlorhydria, are associated with fasting hypergastrinemia and an exaggerated gastrin
meal response. Conversely, antral acidification strongly inhibits gastrin secretion. Locally-released
somatostatin mediates the effects of luminal acidification, inhibiting gastrin secretion.

88. Which of the following statements regarding human gastric acid secretion is/are true?

a. Fasting acid secretion, normally 2 to 5 mEq/h, is due to ambient vagal tone and histamine secretion
b. Truncal vagotomy decreases basal secretion by 80%
c. Histamine2 receptor antagonist administration can decrease basal acid secretion by 80%
d. Fasting acid secretion, normally 5 to 10 mEq/h, is due to circulating levels of gastrin
Answer: a, b, c

Both vagal tone and locally secreted histamine are presumed to be the determinants of basal acid
secretion in humans. Gastrin does not have a role in basal acid secretion in normal individuals.
Parietal cell activation and the resultant acid secretory response is greater to a combination of agonists
than the sum of the responses to the agents used singly. This increase in responsiveness is termed
potentiation. Potentiating interactions are most apparent when the stimulants use different second
messenger systems, for example, acetylcholine and histamine. Conversely, blockade of receptors to one
stimulant also blocks responsiveness to the other agonist. Because of this interaction, blockade of
histamine receptors by agents like cimetidine decreases responsiveness to acetylcholine. Blockage of
acetylcholine release by vagotomy decreases responsiveness to histamine secreted by gastric mast cells.
Both vagotomy and histamine2 receptor antagonists decrease basal acid secretion by approximately 80%.

89. As a meal is emptied from the stomach, gastric acid secretion gradually returns to baseline. Which of
the following statements correctly characterize control of gastric acid secretion?

a. In humans, the most important inhibitory influence on gastrin release is exposure of the gastric
mucosa to luminal acid
b. Acidification of the antral lumen causes reciprocal increases in somatostatin release and decreases in
gastrin secretion
c. Antral distension stimulates gastric acid secretion
d. Acidification of the duodenal bulb inhibits gastric acid secretion
e. Exposure of the duodenum to hyperosmolar solutions inhibits acid secretion
Answer: a, b, d, e

Inhibitory regulation of gastric acid secretion is accomplished by central nervous system, gastric and
small intestinal mechanisms. In humans, the most clearly established gastric inhibitory influence is
suppression of gastrin release by exposure of the antral mucosa to luminal acid. Antral acidification
causes release of gastric mucosal somatostatin which is linked reciprocally to decreases in gastrin
secretion. Antral distension inhibits gastric acid secretion.
The inhibitory phase of gastric acid secretion begins with entry of the products of digestion into the
proximal duodenum. Acidification of the duodenal bulb and exposure of the duodenum to hyperosmolar
solutions and those containing fat potently inhibit acid secretion.

90. Which of the following statements correctly characterizes gastric motor activity associated with
ingestion of a meal?

a. Ingested gastric volumes are accommodated with little increase in pressure by reflex relaxation of the
proximal stomach
b. Receptive gastric accommodation is unaffected by proximal gastric vagotomy
c. In humans, liquid emptying occurs more quickly than solid emptying
d. Gastric emptying of liquids is not affected by proximal gastric vagotomy
Answer: a, c

With ingestion of a meal, increasing gastric volumes are accommodated with little increase in
intragastric pressure by relaxation of the proximal stomach. This process, termed receptive relaxation,
is mediated by a reflex carried by the vagal nerve. After the meal has been ingested, the proximal
stomach is the predominant determinant of the rate of gastric emptying of liquids due to the
gastroduodenal pressure gradient generated by proximal gastric contractions. Liquid emptying occurs
more rapidly than emptying of solids, in part, because liquids are not subject to the sieving actions of
the pylorus.
Truncal and proximal gastric vagotomy abolish receptive relaxation. Following vagotomy, an increased
gastroduodenal pressure gradient is observed and correlates with accelerated liquid emptying. Emptying
of solids is usually not significantly altered by proximal gastric vagotomy.

91. It is widely agreed that the gastric mucosa secretes bicarbonate in addition to acid. Gastric secretion
of bicarbonate is correctly characterized by which of the following statements?

a. Bicarbonate is secreted by chief cells within gastric crypts


b. Gastric bicarbonate secretion is stimulated by acetylcholine
c. Gastric bicarbonate secretion during fasting results in luminal pH above 6 in normal individuals
d. Prostaglandin E2 is a potent stimulant of gastric bicarbonate secretion
Answer: b, d

The gastric cells responsible for bicarbonate secretion are believed to be surface mucous cells facing the
gastric lumen between crypts. Although the total amount of gastric bicarbonate secreted is only a small
fraction of total acid secretion, pH close to neutrality is maintained near the mucosal surface while bulk
luminal pH is highly acidic. Cholinergic agonists, vagal stimulation, and sham feeding all increase gastric
bicarbonate secretion. Prostaglandin E2 and its synthetic derivatives are potent stimulants of
bicarbonate secretion as well. Conversely, indomethacin and other drugs that inhibit prostaglandin
formation decrease mucosal bicarbonate secretion.

92. Gastric mucosal blood flow is regulated by neural, hormonal, and locally active influences. Which of
the following statements correctly characterizes gastric blood flow?

a. Stimulation of sympathetic nerves supplying the stomach is followed by gastric mucosal hyperemia
and increased total gastric blood flow
b. Vagal nerve stimulation is accompanied by decreased gastric mucosal blood flow
c. Stimulants that increase acid secretion increase mucosal blood flow
d. In humans, prostaglandins increase mucosal blood flow at doses that inhibit gastric acid secretion
Answer: c, d

Because the gastric mucosa is metabolically highly active, control of gastric mucosal blood flow is of
great physiologic significance. Almost all stimuli that increase acid secretion also increase gastric blood
flow. A large number of gastrointestinal hormones stimulate gastric blood flow, most because of their
ability to increase acid secretion. Thus, gastrin is a potent stimulant of blood flow, in proportion to its
ability to increase acid secretion. Vagal nerve stimulation has the net effect of increasing mucosal and
total gastric blood flow; sympathetic nerve stimulation is accompanied by opposite effects.
Prostaglandins are important endogenous regulators of gastric blood flow. Prostaglandins of the E class
increase blood flow at doses that suppress acid secretion. Inhibition of cyclo-oxygenase activity by
indomethacin causes a reduction in resting gastric blood flow.

93. Which of the following statements regarding intrinsic factor is/are correct?

a. Intrinsic factor is produced in chief cells located in the gastric fundus


b. Total gastrectomy is followed by folate deficiency due to vitamin malabsorption secondary to intrinsic
factor deficiency
c. Intrinsic factor secretion, like that of acid, is stimulated by gastrin, histamine, and acetylcholine
d. Intrinsic factor deficiency accompanies H pylori-caused antral gastritis
Answer: c

The gastric mucosa is the site of production of intrinsic factor, which is a necessary co-factor for the
absorption of vitamin B12 by the ileal mucosa. Total gastrectomy and atrophic gastritis involving the
proximal oxyntic mucosa are regularly followed by vitamin B12 deficiency, manifest as pernicious
anemia. Acid-secreting parietal cells are the site of intrinsic factor synthesis. Like acid secretion,
intrinsic factor secretion is stimulated by gastrin, histamine, and acetylcholine
94. A 24-year-old woman develops epigastric pain and has a diagnosis of duodenal ulcer confirmed by
esophagogastroduodenoscopy. The patient is in the third month of a pregnancy. The most appropriate
treatment would be:

a. Proximal gastric vagotomy


b. Misoprostol 400 mg b.i.d.
c. Sucralfate 1 gm q.i.d.
d. Cimetidine 400 mg b.i.d.
Answer: c

Cimetidine, ranitidine, famotidine and newer H2 receptor antagonists bind competitively to parietal cell
histamine receptors to produce reversible inhibition of acid secretion. While the pharmacokinetic
profiles of the H2 receptor antagonists differ, when administered at equipotent doses, the agents
produce similar degrees of acid suppression and similar rates of ulcer healing. Cimetidine interacts with
the hepatic microsomal enzyme system and may increase blood levels and pharmacological effects of
drugs that depend upon hepatic metabolism.
Misoprostol, a derivative of prostaglandin E, causes endoscopic healing in 60% of patients at 4 weeks.
The major side effect of misoprostol is diarrhea due to effects upon gut smooth muscle contractility.
Uterine bleeding has been reported in some women using the drug and the agent has potential
abortifacient actions. For this reason, misoprostol is contraindicated in pregnancy.
Sucralfate, the aluminum salt of sulfated sucrose, has virtually no systemic absorption, and for this
reason, is the drug of choice for pregnant patients. When administered at a dose of 1 gm four times
daily, 80% of ulcers will heal by 6 weeks.

95. Helicoabacter pylori has been investigated as a possible etiologic agent in duodenal ulceration.
Which of the following statement(s) regarding H pylori infection in humans is/are correct?

a. H pylori may be isolated from antral gastric mucosa in nearly 100% of patients with active duodenal
ulceration but only 1–2% of normal volunteers
b. H pylori possess cell surface receptors that bind to small intestinal mucous cells
c. Therapeutic regimens for duodenal ulcer that eliminate the organism are associated with lower ulcer
recurrence rates than those in which the organism persists
d. The incidence of the organism in the normal population increases with age
e. Antral gastritis is associated with development of duodenal ulcer
Answer: c, d, e

Helicobacter pylori has received enormous investigative attention in recent years as a possible infectious
cause of peptic ulceration. The evidence that H pylori causes ulcers is substantial but largely inferential.
Antral gastritis is nearly always present in patients with duodenal ulceration. H pylori infestation of
antral mucosa is believed to cause gastritis. While normal small intestinal cells do not permit H pylori
binding, areas of gastric metaplasia are usually demonstrated in the duodenal mucosa immediately
surrounding the ulcer. Resolution of gastritis follows eradication of the organism and drug regimens that
are bactericidal are associated with lower rates of ulcer recurrence than those that have no anti-
bacterial actions.
However, it is clear that not all patients infested with H pylori develop ulceration. Half of patients with
dyspepsia but no ulceration have evidence of H pylori infestation and 20% of healthy volunteers can be
demonstrated to harbor the organism. The incidence of infestation increases with age in the
asymptomatic population.

96. A 40-year-old male undergoes treatment of acute duodenal ulceration with cimetidine 400 mg b.i.d.
and has resolution of symptoms by 6 weeks. The medication is continued as a nocturnal maintenance
dose at the end of a three month treatment course. Recurrent symptoms develop 6 months after initial
diagnosis and repeated endoscopic examination reveals recurrent ulceration. Biopsies of antral mucosa
demonstrate moderate gastritis and the presence of H pylori. Medical management designed to
eradicate H pylori and heal ulceration should include which of the following agents?

a. Cimetidine
b. Bismuth subcitrate
c. Amoxicillin
d. Metronidazole
e. Vancomycin
Answer: a, b, c, d

The observation that H pylori infection has an important role in ulcer pathogenesis has led to
development of antimicrobial therapy for ulceration. Most successful regimens are based on a bismuth
compound (colloidal bismuth subsalicylate or colloidal bismuth subcitrate) plus metronidazole, alone or
in combination with amoxicillin or tetracycline. Bismuth compounds act locally and achieve gastric
concentrations above the MIC for 90% of H pylori isolates. Metronidazole is secreted into the stomach at
high concentration, and the in vivo activity of metronidazole is not diminished by gastric acidity. Triple
therapy with bismuth, metronidazole, and tetracycline or amoxicillin eradicates H pylori in 90% of cases,
compared to 0% eradication with ranitidine. Inclusion of an H2 receptor antagonist or omeprazole has
been reported to increase efficacy of antimicrobial therapy. Currently, antimicrobial therapy has been
recommended for peptic ulcer disease resistant to conventional therapy, including patients with ulcer
relapse while on maintenance therapy and failure to heal in spite of H2 receptor antagonist or
omeprazole therapy.

97. Development of duodenal ulceration is dependent upon gastric acid secretion. Which of the following
statements correctly characterizes acid secretion in duodenal ulcer patients?

a. Groups of duodenal ulcer patients demonstrate decreased basal acid secretion


b. Maximal acid output to histamine averages 40 mEq/h in duodenal ulcer patients, twice that of normal
c. Tissue gastrin levels, on average, are twice normal in patients with active ulceration
d. Exogenously administered somatostatin is ineffective in suppressing acid secretion in patients with
active ulceration
Answer: b

The formation of duodenal ulcer is dependent on gastric secretion of acid and pepsin. As a group,
duodenal ulcer patients have an increased capacity for gastric acid secretion relative to normal
individuals, manifest by increased basal acid secretion, increased acid response to meal ingestion, and
increased responsiveness to histamine stimulation. No definite evidence links abnormalities in
gastrointestinal hormone secretion with increased acid secretion. Tissue gastrin content and circulating
levels of gastrin are normal (excluding Zollinger-Ellison syndrome patients). Secretion of endogenous
somatostatin and responsiveness to exogenously administered somatostatin are likewise normal.

98. A 45-year-old man undergoes proximal gastric vagotomy for treatment of intractable duodenal
ulceration. What physiologic alterations might be anticipated as a consequence of the operation?

a. Reduction of basal acid secretion by approximately 25%


b. Accelerated gastric emptying of liquids
c. Accelerated gastric emptying of solids
d. Fasting hypergastrinemia
e. Postprandial hyperinsulinemia
Answer: b, d

Division of cholinergic vagal fibers directly affects parietal cell acid secretion by reducing stimulatory
input. Basal acid secretion is diminished by approximately 80% and maximal acid output in response to
pentagastrin stimulation is reduced by about 70%. Fasting hypergastrinemia and an exaggerated gastrin
response to meal ingestion is observed due to loss of feedback inhibition of gastrin release and gastrin
cell hyperplasia. Release of pancreatic polypeptide, secretin and cholecystokinin may be decreased.
Proximal gastric vagotomy accelerates gastric emptying of liquids due to a loss of receptive relaxation.
In contrast, gastric emptying of solids is usually not affected by proximal gastric vagotomy.

99. Which of the following statements regarding postoperative rates of recurrent ulcer and dumping
is/are correct?

a. Truncal vagotomy and antrectomy is associated with persistent dumping in 10–15% of patients
b. Recurrent ulceration following truncal vagotomy and pyloroplasty is observed in 25% of patients within
10 years of operation
c. Patients that undergo proximal gastric vagotomy have a risk of recurrent ulcer of 10–15% and a risk of
persistent dumping approximating 1%
d. Recurrent ulceration occurs in 5% of patients that undergo truncal vagotomy and antrectomy
Answer: a, c

Surgical recommendations for treatment of peptic ulceration should be based upon safety, freedom from
long-term postoperative symptoms, and avoidance of recurrent ulceration. Proximal gastric vagotomy
has an operative mortality of less than 1% and a risk of persistent dumping symptoms of approximately
1%. The low incidence of postoperative symptoms is associated with a relatively high risk of recurrent
ulceration, estimated to be 10% to 15% at 5 years postoperatively. After truncal vagotomy and
pyloroplasty, dumping is initially present in 10%, and is persistent or severe in 1%. Recurrent ulceration is
observed in 10% of patients that undergo truncal vagotomy and pyloroplasty. Truncal vagotomy and
antrectomy is associated with the lowest risk of recurrent ulceration, 1–2%, but the greatest incidence of
postoperative dumping symptoms, 10–15%.

100. Which of the following statement(s) is/are correct with regard to pyloric obstruction secondary to
peptic ulceration?

a. Pyloric obstruction is suggested by hypochloremic hyponatremic alkalosis


b. Pyloric obstruction is suggested by hypochloremic hypokalemic alkalosis
c. Approximately 80% of patients with benign gastric outlet obstruction obtain permanent relief of
symptoms by endoscopically-directed balloon dilatation
d. The lifetime risk of pyloric obstruction in peptic ulcer patients is 40%
Answer: b

Repeated episodes of ulceration and healing can lead to scarring and pyloric stenosis. The lifetime risk
of this complication approximates 10%. Gastric outlet obstruction is characterized by the development of
hypochloremic hypokalemic alkalosis due to loss of HCl through vomiting and renal compensatory
mechanisms that conserve H+ at the expense of secreted K+. Although 85% of pyloric stenoses are
technically amenable to balloon dilatation, fewer than 1 in 3 will achieve permanent relief of symptoms
through this means.

101. A 42-year-old man with a recently diagnosed duodenal ulcer develops melena and near-syncope.
After fluid resuscitation, upper gastrointestinal endoscopy is performed. During the examination, a 1 cm
ulcer is noted in the proximal duodenum. A fresh clot is observed within the ulcer and blood is noted to
be oozing around the clot. Optimal therapy would consist of which of the following?

a. Angiographic embolization of the gastroduodenal artery


b. Irrigation of the clot followed by endoscopic application of a heat probe
c. Transfusion and intravenous cimetidine
d. Angiographic infusion of vasopressin into the gastroduodenal artery
e. Transfusion and oral omeprazole
Answer: b

The ability to visualize bleeding duodenal ulcers endoscopically has led to attempts to treat hemorrhage
endoscopically. Thermal coagulation may be achieved by bipolar electrocoagulation or by direct
application of heat through a probe. An NIH Consensus Development Conference has recommended
endoscopic hemostatic therapy in selected patients. Hematemesis, age over 60, and serious medical co-
morbidity are clinical features that mandate endoscopic therapy. Rebleeding during hospitalization and
the endoscopic findings of visible vessel, oozing, or bleeding associated with an adherent clot are other
indications for endoscopic hemostasis. Operative intervention is appropriate for massive hemorrhage
leading to shock or cardiovascular instability, prolonged blood loss requiring continuing transfusion,
recurrent bleeding during medical therapy or after endoscopic therapy, and recurrent hemorrhage
requiring hospitalization. Operative therapy should consist of duodenotomy with direct ligation of the
bleeding vessel within the ulcer base followed by a procedure to permanently reduce acid production.

102. A 50-year-old patient has undergone truncal vagotomy and antrectomy with Billroth II
reconstruction two years ago. The patient now complains of recurrent postprandial pain, nausea, and
vomiting. Endoscopic examination reveals bile in the stomach; endoscopic biopsies demonstrate
histologic evidence of moderately severe gastritis. No other endoscopic abnormalities are noted.
Appropriate therapy could include:
a. Octreotide administration
b. Total gastrectomy
c. Conversion of Billroth II gastrojejunostomy to Billroth I gastroduodenostomy
d. Conversion of Billroth II gastrojejunostomy to Roux-en-Y gastrojejunostomy
e. Roux-en-Y hepaticojejunostomy
Answer: d

Symptoms related bile reflux gastritis occur transiently in 10% to 20% of patients after truncal vagotomy
and resection or drainage. Symptoms persist in only 1% to 2%. No completely satisfactory solution to bile
reflux gastritis exists. Medicinal and dietary treatments have not been proven to be beneficial.
Operative diversion of biliary secretions away from the gastric mucosa by construction of a Roux-en-Y
gastrojejunostomy with an intestinal limb of 50 to 60 cm has been widely reported. The procedure
eliminates bilious vomiting in nearly 100% of patients, but pain persists in up to 30%, and 20% develop
delayed gastric emptying as a result of the procedure.

103. A 50-year-old male with a 2 year history of duodenal ulceration develops sudden, severe epigastric
pain 4 hours prior to evaluation. Physical examination reveals T 101° F, pulse 80, BP 125/90, diminished
bowel sounds, and abdominal muscular rigidity. An upright chest x-ray reveals pneumoperitoneum. At
laparotomy, an anterior perforation in the first portion of the duodenum is observed. Optimal treatment
would include:

a. Omental patch of the perforation followed by truncal vagotomy and antrectomy after 8 weeks
b. Omental patch of the perforation followed by truncal vagotomy and pyloroplasty after 8 weeks
c. Omental patch of the perforation followed by chronic cimetidine administration
d. Omental patch of the perforation plus proximal gastric vagotomy
e. Omental patch of the perforation only
Answer: d

Simple omental patching of a perforation in patients with chronic ulcer disease does not yield
satisfactory long-term results. Up to 80% of patients so treated develop recurrent ulceration and 10%
develop secondary complications. A definitive ulcer operation should be performed during the initial
laparotomy if the following circumstances apply: 1) there has been no preoperative shock, 2) the
perforation has been present for less than 48 hours, and 3) no life-threatening medical co-morbidity
exists. Omental patching of the perforation combined with proximal gastric vagotomy is a preferred
approach because it combines safety, freedom from disabling postoperative symptoms, and a low rate of
recurrent ulceration.

104. Which of the following clinical circumstances have been identified as predisposing factors for the
development of stress ulceration?

a. Intraperitoneal sepsis
b. Hemorrhagic shock
c. Isolated tibial fracture
d. 50% total surface area second degree burn
e. Adult respiratory distress syndrome
Answer: a, b, d, e

Several risk factors or predisposing clinical conditions have been identified for stress ulceration. Specific
risk factors include adult respiratory distress syndrome, multiple trauma, major burn of over 35% of body
surface area, oliguric renal failure, large transfusion requirements, hepatic dysfunction, hypotension,
prolonged surgical procedures, and sepsis from any source. A direct correlation has been shown between
acute upper gastrointestinal hemorrhage and the severity of critical illness.

105. Type I gastric ulcers are located in the gastric body, usually along the lesser curvature. Which of
the following statements correctly characterize type I gastric ulcers?

a. Normal to low acid secretion


b. Associated duodenal ulceration
c. High frequency of blood group A
d. Associated hypergastrinemia frequent
Answer: a, c

Gastric ulcers are divided into categories based on their location and gastric acid secretory status. A
type I gastric ulcer is an ulcer in the body of the stomach, usually along the lesser curvature, associated
with large volumes of secretion with a low to normal acid output. Type I ulcers are not associated with
duodenal, pyloric, or prepyloric mucosal abnormalities. There is a slight predominance of patients with
blood group A in this type of gastric ulcer.
Type II gastric ulcer is located in the body of the stomach in combination with a duodenal ulcer. These
patients are usually acid hypersecretors. About 23% to 25% of gastric ulcers are type II. A type III gastric
ulcer is characterized as a prepyloric ulcer and accounts for about 23% of lesions. Patients with this
lesion are typically acid hypersecretors.
Type IV gastric ulcer occur high on the lesser curvature near the gastroesophageal junction. In the
United States, the incidence of type IV gastric ulcer is less than 10%.

106. Which of the following statement(s) is/are correct regarding gastric ulcers greater than 3 cm in
size?

a. Giant gastric ulcers occur in 30–40% of cases along the greater curvature
b. The risk of malignancy increases with increasing size of the ulcer
c. The treatment of choice for giant gastric ulcer is resection to include the ulcer
d. Giant gastric ulcer is a complication of intraarterial hepatic chemotherapy
Answer: b, c

A giant gastric ulcer is defined as an ulcer whose diameter is 3 cm or greater. The lesser curvature is the
most common site, with only 3% to 4% occurring along the greater curvature. Gastric ulcers often
penetrate into contiguous structures such as spleen, pancreas, liver, and transverse colon, and can be
falsely diagnosed as a nonresectable malignancy, despite normal biopsy results. Most reports cite an
incidence of malignancy ranging from 6% to 30%, increasing with the size of the ulcer. Because of the
high likelihood of complications from giant gastric ulcer, early operation is the treatment of choice. The
operation of choice is resection, including the ulcer. If the ulcer has penetrated adjacent structures and
cannot be dissected free, the stomach wall can be incised circumferentially, leaving the ulcer intact and
behind, and the gastric resection completed.

107. With regard to benign gastric ulceration, the most common location of disease is which of the
following?

a. Along the greater curvature


b. Immediately distal to the esophagogastric junction along the lesser curvature
c. In the area of the incisura angularis along the lesser curvature
d. Within the gastric antrum
Answer: c

Gastric ulcers can occur anywhere in the stomach, although they usually present on the lesser curvature
near the incisura angularis. About 60% are located at or slightly above the angularis. Fifteen percent to
23% of gastric ulcers are within the gastric antrum and 10% are high on the lesser curvature. Only 5% of
gastric ulcers are found on the greater curvature. In addition, 97% of all gastric ulcers occur within 2 cm
of the junctional zone between fundic and antral mucosa. Gastric ulcers appear at different distances
from the pyloric sphincter because the antrum extends for variable (2 to 16 cm) distances from the
pylorus. It is interesting that with increasing age this junctional zone moves proximally along the lesser
curvature, as does the incidence of gastric ulcer.

108. Which of the following statement(s) regarding gastric mucosal defense is/are correct?

a. Gastric mucus, produced by the surface epithelial cells, forms an unstirred layer over the gastric
surface
b. Gastric parietal cells produce a bicarbonate-rich fluid
c. Production of gastric bicarbonate is stimulated by prostaglandins and inhibited by non-steroidal
antiinflammatory drugs (NSAIDs)
d. Gastric mucus provides substantial buffering capacity that maintains near-neutrality near the
epithelial surface
Answer: a, c

Gastric mucus is produced by the surface epithelial cell. In addition to serving as a lubricant, mucus also
enhances mucosal defense by forming an unstirred layer overlying the epithelial surface. In humans, the
thickness of this gel layer has been reported to be about 500 µm. The vacuoles containing mucus in the
cytosol of the surface epithelial cell are released by cholinergic stimulation and prostaglandins.
Synthesis and release are inhibited by aspirin-like compounds that inhibit cyclooxygenase. Gel mucus
retards proton mobility by a factor of only three or four, a degree that is inadequate to maintain a near
neutral pH at the apical membrane of the surface epithelial cell.
In addition to producing mucus, the gastric surface epithelial cells secrete a bicarbonate-rich fluid. The
amount of bicarbonate secreted is about 5% to 10% of the amount of acid that the same surface of
mucosa can produce. As with mucus, bicarbonate secretion is stimulated by cholinergic agents and
prostaglandins and inhibited by cyclooxygenase inhibitors.
The release of bicarbonate into the gel mucus provides a significant mucosal defense by maintaining a
near neutral acid-base milieu at the apical membrane of the surface epithelial cells. When the luminal
pH is around 3.0, the apical membrane of the surface epithelial cell may be exposed to a pH of about
5.0.

109. A 35-year-old smoker is involved in a house fire and receives a 45% total surface area burn. One half
of the burned surface appears to be third degree. On the third post-burn day, the patient is noted to
have bloody drainage from a nasogastric tube and a decrease of 5% in his hematocrit. Appropriate
management should include which of the following?

a. Urgent upper gastrointestinal contrast study to delineate site of bleeding


b. Immediate selective arteriography via the left gastric artery to diagnose and treat presumed stress
ulceration
c. Urgent esophagogastroduodenoscopy to diagnosis the cause of bleeding
d. Urgent intravenous infusion of vasopressin at 0.2–0.4 IU/min
Answer: c

Patients who have sustained a major thermal burn of 35% or more of their body surface area are at a
predictably high risk for the development of gastric erosions and hemorrhage. Endoscopy has
demonstrated that gastric erosions are present in 93% of these patients, whereas the occurrence of
severe acute upper gastrointestinal hemorrhage in severely burned patients ranges between 25% to 50%.
At least 60% of patients at risk develop stress erosions within 1 to 2 days after the precipitating event.
Painless upper gastrointestinal bleeding may be the only clinical sign. The onset of hemorrhage is often
delayed, usually occurring 3 to 10 days after the onset of the primary disease.
Esophagogastroduodenoscopy is the diagnostic modality of choice to confirm the diagnosis and to
differentiate stress erosion from other sources of upper gastrointestinal hemorrhage. Correct
identification of the bleeding source is made in greater than 90% of instances. If endoscopy is not
diagnostic, visceral angiography through selective catheterization of the left gastric or splenic vessels
may provide information regarding the primary vessel supplying the bleeding site. In contrast, barium
examinations are usually of little value, due to the superficial nature of stress erosions, and in fact may
be detrimental by interfering with the interpretation of subsequent arteriography.

110. Agents demonstrated to have an efficacy of greater than 90% for prophylactic treatment of stress
ulceration include which of the following?

a. Antacids
b. H2 receptor antagonists
c. Sucralfate
d. Misoprostil
Answer: a, b, c

The hourly administration of antacid (30 to 60 mL) by nasogastric tube, maintaining the gastric luminal
fluid at pH above 3.5, has proven to be effective prophylaxis. In a study of 100 seriously ill patients who
were randomly assigned to receive placebo or antacid prophylaxis, bleeding was detected in 25% of
patients given no prophylaxis, compared to 4% of patients given antacids through the nasogastric tube. In
a review of data derived from 16 prospective trials, when overt bleeding manifested by melena,
hematemesis, or transfusion requirement was used as the minimum criterion, there was no significant
difference in risk of bleeding, comparing antacids and cimetidine.
Continuous infusions of any of the H2-receptor antagonists provides more consistent maintenance of an
intraluminal gastric pH of greater than 3.5 than do the standard intermittent-infusion regimens.
Advantages of continuous infusion of these agents include a potential reduction in toxicity, decreased
pharmacy costs and nursing duties, and possible enhancement of therapeutic benefit. Controlled trials
suggest that sucralfate, 1 g every 6 hours, may be as effective as antacids or cimetidine
prophylactically. In 100 critically ill patients, bleeding occurred in 6% of patients receiving antacids or
cimetidine, while none of the 34 patients on sucralfate bled.
Given exogenously, natural or synthetic prostaglandins of the E, F, and I series inhibit gastric acid
secretion. One group compared the efficacy of 15(R)-15 dimethyl PGE2 given at antisecretory doses to
antacids and found that stress-related bleeding occurred in 50% of patients given the synthetic
prostaglandin derivative, compared to only 14% of patients receiving antacids.

111. A 45-year-old male presents with symptoms of epigastric pain, worsened with ingestion of food.
Physical examination is normal. Upper abdominal ultrasonography is unremarkable. Contrast radiography
reveals a 2 cm ulcer in the gastric fundus along the lesser curvature. Therapy with omeprazole 20 mg per
day is begun but symptoms persist 3 weeks later. Appropriate management includes which of the
following?

a. Increase in omeprazole dose to 40 mg per day


b. Addition of sucralfate 1 gm every 8 hours
c. Addition of cimetidine 200 mg b.i.d.
d. Esophagogastroduodenoscopy with biopsy of ulceration
Answer: d

About 5% of ulcers appearing radiographically benign are malignant} Gastroscopy is the most reliable
method of distinguishing benign and malignant gastric ulcer, with an accuracy of more than 97% if
multiple biopsies and brushings for cytology are performed. Clinical features prompting early endoscopic
evaluation include major weight loss, symptoms of gastric outlet obstruction, a palpable abdominal
mass, and stool hemoccult positivity or blood loss anemia. Endoscopic features that suggest malignancy
include an exophytic mass, abnormal or disrupted mucosal folds, necrotic ulcer crater, bleeding from
the edge of the ulcer crater, a stepwise depression of the ulcer edge, heaped-up margins, or small
extensions of the ulcer that blur a portion of the ulcer wall. If initial biopsies do not demonstrate
malignant cells but the endoscopic appearance strongly suggests that underlying the ulcer is a
carcinoma, repeat endoscopy with deeper biopsies should be undertaken.

112. A 52-year-old woman is hospitalized with acute upper gastrointestinal hemorrhage. Endoscopic
examination reveal a 2.5 cm ulcer in the area of the incisura angularis. The remainder of the endoscopic
examination is normal. Continued bleeding requires operative therapy. Optimal therapy consists of
which of the following?

a. Gastrotomy with oversewing of the bleeding site


b. Distal gastrectomy including the area of ulceration
c. Proximal gastric vagotomy and oversewing of the bleeding ulcer
d. Truncal vagotomy, pyloroplasty, and oversewing of the bleeding ulcer
Answer: b

A distinction should be made among the different types of gastric ulcer in selecting the most appropriate
operative procedure, because treatment varies according to location, coexistent duodenal ulcer disease,
and acid secretory status. The elective operation of choice for a type I benign gastric ulcer is a distal
gastrectomy with gastroduodenal anastomosis. Gastrojejunostomy is an acceptable alternative. The
ulcer should be included in the antrectomy specimen. The operative mortality rate associated with this
procedure is 2% to 3%, the recurrence rate is 3%, and a good to excellent clinical result can be
anticipated in more than 90% of patients. The addition of truncal vagotomy does not appear to diminish
the recurrence rate.
Definitive treatment for hemorrhage is accomplished by a procedure designed to control bleeding in
addition to preventing recurrent ulceration. An antrectomy, which includes the ulcer with
gastroduodenostomy, is considered the procedure of choice for surgical treatment of this complication.
The quoted operative mortality rates in this setting range from 10% to 40%.
113. A 25-year-old man is involved in an automobile accident with resultant injuries including bilateral
closed femur fractures, left pulmonary contusion, and closed head injury. On post-injury day 4,
significant upper gastrointestinal hemorrhage begins. Endoscopic examination reveals an area of
confluent ulceration with bleeding in the gastric fundus. Endoscopic hemostasis fails. Appropriate
immediate management includes:

a. Lavage of gastric contents with iced saline


b. Urgent total gastrectomy
c. Selective arterial infusion of vasopressin via the left gastric artery
d. Insertion of Sangstaken-Blakemore balloon
Answer: c

Initial efforts to control gastric hemorrhage consist of gastric lavage using warmed saline. Lavage serves
to fragment existing clots and to remove any pooled blood, reducing fibrinolysis at bleeding sites. Over
80% of patients who present with upper gastrointestinal hemorrhage stop bleeding using this approach.
Definitive treatment of ongoing acute active stress bleeding by antacids is largely unsuccessful.
Administration of H2-receptor blocking agents once active gastrointestinal bleeding has commenced is
also usually ineffective as a definitive form of therapy.
The endoscope has become the preferred therapeutic as well as diagnostic instrument with
electrocautery and laser photocoagulation capabilities. If endoscopic therapy fails, angiography offers an
additional means for the control of bleeding by selective infusion of vasopressin into the splanchnic
circulation via the left gastric artery. Vasopressin is administered by continuous infusion through the
catheter at a rate of 0.2 to 0.4 IU/min for a maximum of 48 to 72 hours.
About 10% to 20% of patients with acute stress ulcers continue to bleed or have recurrent bleeding
despite these measures. In these patients, total gastrectomy has a mortality ranging from 17% to 100%.
In general, operative mortality rates for acute stress-induced hemorrhage range from 30% to 60%
regardless of the surgical procedure undertaken.

114. Which of the following statements regarding gastroplasty and gastric bypass for morbid obesity
is/are correct?

a. Horizontal gastroplasty techniques that rely on a single horizontal application of a stapling device are
associated with weight loss “failure” rates of 40% to 70%
b. Gastric bypass is followed by progressive weight loss over a period of 36 months
c. Gastric bypass is associated with a “failure” of weight loss in 10 to 15% of patients
d. With three superimposed applications of a stapling device, gastric bypass staple line dehiscence
occurs in less than 2%
Answer: a, c, d

Horizontal gastroplasties include a single application of a 90-mm stapling device without suture
reinforcement of the “stoma’’ between upper and lower gastric pouches or a double application of
staples with either a central or lateral prolene-reinforced stoma. The failure rates (loss of less than 40%
excess weight) for horizontal gastroplasty procedures ranges from 40% to 70%. The vertical banded
gastroplasty (VBGP) is a procedure in which a stapled opening is made in the stomach with the stapling
device 5 cm from the cardioesophageal junction. Two applications of a 90-mm stapling device are made
between this opening and the angle of His, and a 1.5 5 cm strip of polypropylene mesh is wrapped
around the stoma on the lesser curvature and sutured to itself.
Gastric bypass can be performed with placement of staples in a vertical or horizontal direction; the
vertical direction is preferred because there is less risk of gastric pouch devascularization or splenic
injury. With three superimposed applications of a 90-mm stapler, the incidence of staple line disruption
has been less than 2%.
Roux-en-Y gastric bypass has significantly better weight loss than VBGP. Although 10% to 15% of patients
fail gastric bypass, weight loss seems to remain stable in most patients over 5 years or more after
surgery.

115. Severe obesity is associated with a large number of associated problems that form the basis of the
term morbid obesity. Documented causes of excess mortality in severely obese patients include:

a. Coronary artery disease


b. Hypertension
c. Adult-onset diabetes mellitus
d. Obesity hypoventilation and sleep apnea
e. Pulmonary embolization
Answer: a, b, c, d, e

Morbid obesity is arbitrarily defined as 100 pounds above ideal body weight as defined by actuarial
tables. Premature death is much more common in the severely obese; morbidly obese men aged 25 to 34
years have a 12-fold increase in mortality relative to normal. Causes of early mortality include coronary
artery disease, hypertension, impaired ventricular function, diabetes mellitus, sleep apnea and other
hypoventilation syndromes, pulmonary embolization, and necrotizing soft tissue infections.

116. Jejunoileal bypass was formerly performed as a weight reduction procedure. The operation has now
been abandoned due to the development of serious long-term complications associated with the
procedure. Which of the following statements correctly characterize results following jejunoileal bypass?

a. Kidney stones occur with increased frequency due to increased absorption of pyruvate from the colon
b. The most serious complication of jejunoileal bypass is development of cirrhosis due to protein calorie
malnutrition
c. Bacterial overgrowth in the bypassed segment can be treated with oral vancomycin
d. Rapid weight loss following jejunoileal bypass is associated with development of gallstones
Answer: b, d

Jejunoileal bypass is associated with a number of early and late complications. Malabsorption of bile
salts, coupled with rapid weight loss, significantly increases risk of gallstone development. Multiple
kidney stones result from excessive absorption of oxylate from the colon where oxylate is ordinarily
chelated with calcium. Malabsorption results in severe diarrhea, electrolyte abnormalities, metabolic
acidosis and anemia. Bacterial overgrowth in the bypassed intestinal segment coupled with protein
malabsorption is postulated to be responsible for development of cirrhosis, the most serious
complication of jejunoileal bypass. Bacterial overgrowth can be temporarily suppressed by
metronidazole. Development of hepatic dysfunction is an indication for reversal of the bypass.

117. Which of the following statements is correct with regard to gastric bypass for obesity?

a. Rapid weight loss following successful gastric bypass for obesity is associated with an increased risk of
developing cholelithiasis
b. Marginal ulcer develops in 25% of gastric bypass patients
c. Vitamin B12 deficiency is a potential complication of gastric bypass due to gastric mucosal atrophy
d. Anastamotic leak after gastric bypass is often heralded by bradycardia
Answer: a

The most serious complication after gastric bypass for obesity is anastamotic dehiscence. Leak is
presumed to occur secondary to gastric necrosis due to ischemia from staple line application or short
gastric vessel ligation. Affected patients may have little pain, with tachycardia, tachypnea and fever as
the only manifestations. Physical examination in morbidly obese patients with peritonitis is unreliable.
Marginal ulcers occur in only 10% or less of gastric bypass patients and respond to H2 receptor
antagonists. Rarely, polyneuropathy has been noted after gastric bypass, usually in association with
intractable vomiting and protein calorie malnutrition. Vitamin B12 deficiency has been noted after
gastric bypass due to decreased acid digestion of vitamin B12 in food; monthly B12 supplementation
should be routine. Cholelithiasis occurs in about one third of morbidly obese patients and gallstone
formation is accelerated in the early postoperative period by the effects of rapid weight loss.

118. With regard to operative treatment of gastric carcinoma, which of the following statements is/are
correct?

a. Resectional margins of 2 cm are necessary to prevent recurrence due to intramural metastases


b. Prophylactic splenectomy has been shown to improve outcome in similarly staged patients
c. Extended lymphadenectomy including nodes along the aorta and esophagus has not been shown to
improve survival in North American trials
d. Long-term survival is rare if adjacent organs must be resected to achieve local control
Answer: c, d

In gastric cancer, microscopic involvement of the resection margin by tumor cells is associated with poor
prognosis. In contrast to colon cancer, gastric cancer frequently demonstrates extensive intramural
spread. Retrospective studies suggest that a line of resection 6 cm from the tumor mass is necessary to
ensure a low rate of anastamotic recurrence.
The value of extended lymphadenectomy in the treatment of gastric adenocarcinoma is controversial.
The largest favorable experience has been reported by Japanese surgeons, where retrospective studies
have suggested an improvement of approximately 10%, stage for stage, for patients with advanced
disease. The benefits of extensive lymphadenectomy have not been confirmed in western countries.
Histologically positive lymph nodes are frequently present in the splenic hilum and along the splenic
artery, and routine splenectomy has been practiced in some centers. Prophylactic splenectomy has not
been demonstrated to improve outcome for similarly staged patients. Resection of adjacent organs may
be required for local control if direct invasion has occurred. In this circumstance, operative morbidity is
increased, and long-term survival is rare.

119. Which of the following statement(s) characterizing gastric lymphoma is/are correct?

a. More than half of gastrointestinal lymphomas occur in the stomach


b. The peak incidence of gastric lymphoma is in the 2nd and 3rd decade
c. Endoscopic biopsy is positive diagnostically in 90% of cases
d. Gastric perforation occurs in 40% of patients treated with cytolytic agents instead of gastrectomy
Answer: a, c

The stomach is the site of more than half of gastrointestinal lymphomas and is the most common organ
involved in extranodal lymphomas. Gastric lymphoma is distinctly uncommon in children and young
adults. The peak incidence is in the sixth and seventh decades. Radiologic findings are similar to those
noted for adenocarcinoma. Endoscopic examination has become the diagnostic method of choice.
Endoscopic biopsy, combined with endoscopic brush cytology, provides positive diagnosis in some 90% of
cases. When gastric lymphoma is first diagnosed by endoscopic means, evidence of systemic disease
should be sought. CT of chest and abdomen (to detect lymphadenopathy), lymphangiography, bone
marrow biopsy, and biopsy of enlarged peripheral lymph nodes may all be appropriate.
A multimodality treatment program is used in most centers for primary gastric lymphomas, with
gastrectomy as the first step in the therapeutic strategy. Increasing numbers of patients are treated with
chemoradiation therapy alone. The risk of hemorrhage or perforation was frequently alluded to in the
past and has probably been overstated. The risk of perforation in primary gastric lymphomas that are
treated with cytolytic agents in unresected patients approximates 5%.

120. A patient with gastric adenocarcinoma undergoes subtotal gastrectomy. Pathological examination
reveals that the tumor penetrates to the serosa. Regional lymph nodes are not involved. Distant
metastases are not detected. What is the correct tumor stage and 5-year survival rate?

a. Stage I: 90% 5-year survival


b. Stage II: 45% 5-year survival
c. Stage III: 15% 5-year survival
d. Stage II: 15% 5-year survival
e. Stage III: 2% 5-year survival
Answer: b

For early lesions of the antrum or middle stomach, distal subtotal gastrectomy including 80% of the
stomach provides satisfactory 5-year survival without increases in operative morbidity. Proximal gastric
lesions or larger middle stomach lesions may require total gastrectomy or esophagogastrectomy to
encompass the tumor. Regardless of the extent of gastric resection, patients with more advanced tumors
fare poorly because of the increased likelihood of lymphatic and hematogenous spread.
The TNM system is shown in Table 25-2. Five-year survival for stage I patients (in situ carcinoma) is close
to that of the normal population. In stage II, 5-year survival approximates 45%, while 15% of stage III
patients survive 5 years. Prolonged survival with systemic metastases in negligible.

121. Which of the following conditions is considered to increase the risk of gastric cancer?

a. Pernicious anemia
b. Prior partial gastrectomy
c. Gastric hyperplastic polyps
d. Gastric adenomatous polyps
Answer: a, b, d

The risk of developing gastric cancer is greater in stomachs that harbor adenomatous polyps. The risk of
developing cancer has been estimated at 10% to 20% and is greatest for polyps more than 2 cm in
diameter. Hyperplastic polyps, while common in the normal population, do not have malignant
potential. Gastric malignancy is increased in individuals with chronic gastritis associated with pernicious
anemia. When pernicious anemia has been present for 5 years, the risk of gastric malignancy is twice
that of age-matched controls. An increased risk of gastric carcinoids also exists in patients with
pernicious anemia, presumably due to the effects of long-standing hypergastrinemia. A 3-fold increased
risk of gastric cancer also exists in patients that have previously undergone partial gastric resection.
Postgastrectomy cancer is a long-term concern with increased incidence of malignancy not observed
until 15 years postoperatively.

122. A 55-year-old male is evaluated because of symptoms of epigastric pain and anorexia. Physical
examination is normal except for guaiac positivity of stool. Upper endoscopic examination reveals a 1.5
cm ulcer along the lesser curvature of the stomach proximal to the incisura angularis. Optimal
management consists of which of the following:

a. Sucralfate 1 gm q.i.d. for 8 weeks


b. Endoscopic biopsy of the ulcer rim
c. Endoscopic cautery of the ulcer base
d. Endoscopic biopsy of the ulcer base
e. Misoprostol 400 mg b.i.d. for 8 weeks
Answer: b

The symptoms produced by gastric cancer and benign gastric ulcer are non-specific and often similar.
Pain is present in 70% of patients with gastric cancer and is usually constant, non-radiating, and not
improved by food ingestion. Physical examination is usually normal in patients with early gastric cancer.
Guaiac positivity is noted in one third.
Fiberoptic endoscopy is the definitive diagnostic method. Although the endoscopic appearance of gastric
ulcers may suggest benign or malignant origins, definite distinction can only be made by gastric biopsy.
Accurate diagnosis of gastric cancer can be made in 95% of cases if multiple biopsies are obtained from
the ulcer rim. Biopsy of the ulcer base will more frequently reveal necrotic material.

123. Which of the following statement(s) regarding gastric leiomyosarcoma is/are correct?

a. Leiomyosarcomas occur with peak frequency in the 2nd and 3rd decades
b. The primary histological indicator of aggressive behavior is the number of mitoses per microscopic
field
c. Leiomyosarcomas are usually radiosensitive
d. Lymphadectomy is not indicated during resection because metastases are usually hematogenous
Answer: b, d

Leiomyosarcomas occur with equal frequency in both sexes in the sixth and seventh decades of life. The
tumor frequently may have prominent extraluminal growth and attain large size before causing
symptoms. Leiomyosarcomas must be differentiated from their benign counterparts, leiomyomas.
Grossly, the tumors are firm, gray-white masses; a pseudocapsule separating tumor from normal smooth
muscle may occasionally be present. When the tumors reach a large size, central necrosis is common.
Leiomyosarcomas are usually graded histologically, with the frequency of mitotic figures the prime
indicator of aggressive behavior. Lesions with more than 5 to 10 mitoses per 10 high-power fields
demonstrate increased metastasis.
Intraperitoneal sarcomatosis is frequent, as is local recurrence after resection. Metastasis occurs by way
of the hematogenous route, and thus hepatic involvement is common. Lymphatic metastasis is observed
in less than 10% of patients. Negative surgical margins must be ensured histologically, but
lymphadenectomy is not indicated because of the low frequency of lymphatic metastasis.
Leiomyosarcomas are not radiosensitive, and chemotherapy has not been shown to improve survival.

You might also like